Site Loader

Содержание

5.6 Расчёт фнч первого порядка

Если порядок фильтра нечетный, то одно из звеньев должно быть первого порядка. Схема подобного звена нижних частот приведена на рис. 5.3. Звено представляет собой пассивный RC-фильтр первого порядка, выход которого соединён с входом буферного каскада – в данном случае операционного усилителя в неинвертирующем включении. Данное звено может иметь большой коэффициент усиления и в том случае, если требуется получить значительный коэффициент усиления всего фильтра, на звено первого порядка можно возложить основную функцию усиления.

Рис. 5.3. Фильтр нижних частот первого порядка

Частотная передаточная функция фильтра определяется выражением

.

Для расчёта элементов фильтра необходимо выполнить следующие действия.

1) Выбрать коэффициент усиления звена K.

2) Из табл. 5.1…5.8 определить передаточную функцию звена первого порядка, рассчитать полюс передаточной функции фильтра и собственную нормированную частоту Qp, характеризующую полюс.

В данном случае передаточная функция имеет вид

.

Здесь K – коэффициент усиления звена, С – коэффициент нормированного полинома фильтра нижних частот.

3) Выбрать номинальное значение ёмкости С1, близкое к значению

,

взятому в мкФ.

4) Вычислить значения сопротивлений

а) Если K > 1

,

,

.

б) Если K = 1, то сопротивление R1 определяется так же, как в пункте а, сопротивление R2 отсутствует, а R3 заменяется короткозамкнутой цепью, т. е. схема работает на повторителе напряжения.

5) Выбрать номинальные значения сопротивлений как можно ближе к вычисленным значениям и реализовать звено первого порядка в соответствии со схемой, показанной на рис. 5.3.

6 Расчёт активных rc-фильтров верхних частот

6.1 Передаточная функция фвч

Передаточную функцию ФВЧ с частотой среза с можно получить из передаточной функции нормированного ФНЧ с помощью замены переменной p на с/p. Следовательно, передаточная функция звена второго порядка фильтра верхних частот Баттерворта или Чебышева будет иметь вид

,

где с – частота среза, В и С – нормированные коэффициенты звена нижних частот второго порядка, K – коэффициент усиления звена.

Передаточная функция звена первого порядка

,

где С – нормированный коэффициент звена нижних частот первого порядка.

Приведённые формулы означают, что при проектировании активных RC-фильтров верхних частот можно использовать нормированные передаточные функции ФНЧ. В этом случае ФВЧ заменяют эквивалентным ФНЧ изображения и производят преобразование частот. Собственная частота ФНЧ изображения

.

Нормированная частота задержания ФНЧ изображения определяется аналогично

.

Коэффициенты передачи или ослабления для ФНЧ изображения берутся такими же, как для фильтра верхних частот. Для ФНЧ изображения выбирают тип аппроксимации, находят порядок фильтра

n, из табл. 5.1…5.8 находят коэффициенты нормированного полинома знаменателя передаточной функции. Затем определяют собственные нормированные частоты звеньев фильтра изображения pн и добротности Qpн.

После указанных расчётов выбирают конкретные электрические схемы звеньев, реализующие фильтр, и рассчитывают параметры элементов звеньев (номинальные значения пассивных элементов).

О БЕДНОЙ ПИЩАЛКЕ ЗАМОЛВИТЕ СЛОВО. Расчет фильтров для ВЧ головок ( пищалок ).

О БЕДНОЙ ПИЩАЛКЕ ЗАМОЛВИТЕ СЛОВО

А.И.Шихатов 2003

      Традиционно раздел полос СЧ и ВЧ (или мидбас-ВЧ) производят пассивными кроссоверами (разделительными фильтрами). Это особенно удобно при использовании готовых компонентных наборов. Однако, хотя характеристики кроссоверов и оптимизированы для данного комплекта, они не всегда удовлетворяют поставленной задаче.
      Рост индуктивности звуковой катушки с частотой приводит к увеличению импеданса головки. Причем индуктивность эта у «среднестатистического» мидбаса составляет 0,3-0,5 мГн, и уже на частотах 2-3 кГц импеданс возрастает практически в два раза. Поэтому при расчете пассивных кроссоверов применяют два подхода: используют в расчетах реальное значение импеданса на частоте раздела или вводят цепи стабилизации импеданса (компенсаторы Цобеля). Об этом уже написано немало, поэтом не будем повторяться.

      У пищалок стабилизирующие цепи обычно отсутствуют. При этом исходят из того, что рабочая полоса частот невелика (две-три октавы), а индуктивность незначительна (обычно менее 0,1 мГн). Вследствие этого рост импеданса невелик. В крайнем случае, увеличение импеданса компенсируют резистором сопротивлением 5-10 Ом, включенным параллельно пищалке.
      Однако все не так просто, как кажется на первый взгляд, и даже такая скромная индуктивность приводит к любопытным последствиям. Проблема заключена в том, что пищалки работают совместно с фильтром ВЧ. Независимо от порядка в нем имеется емкость, включенная последовательно с пищалкой, и она образует с индуктивностью звуковой катушки колебательный контур. Частота резонанса контура оказывается в полосе рабочих частот пищалки, и на АЧХ возникает «горб», величина которого зависит от добротности этого контура. В результате неизбежна окраска звучания. В последнее время появилась немало моделей пищалок высокой чувствительности (92 дБ и выше), индуктивность которых достигает 0,25 мГн. Поэтому вопрос согласования пищалки с пассивным кроссовером приобретает особую остроту.
      Для анализа использовалась среда моделирования Micro-Cap 6.0, но те же результаты можно получить и с помощью других программ (Electronic WorkBench, например). В качестве иллюстраций приведены только наиболее характерные случаи, остальные рекомендации даны в конце статьи в виде выводов. В расчетах использовалась упрощенная модель пищалки, учитывающая только ее индуктивность и активное сопротивление. Данное упрощение вполне допустимо, поскольку резонансный пик импеданса большинства современных пищалок невелик, а частота механического резонанса подвижной системы находится за пределами рабочей полосы частот. Учтем также, что АЧХ по звуковому давлению и АЧХ по электрическому напряжению — две большие разницы, как говорят в Одессе.
      Взаимодействие пищалки с кроссовером особенно хорошо заметно у фильтров первого порядка, характерных для недорогих моделей (рисунок 1):


Рисунок 1

      Видно, что даже при индуктивности 0,1 мГн имеется выраженный пик в области частот 7-10 кГц, придающий звучанию характерную «хрустальную» окраску». Увеличение индуктивности смещает резонансный пик в область более низких частот и увеличивает его добротность, что приводит к заметному «цыканью». Побочное следствие увеличение добротности, которое можно обратить на пользу — увеличение крутизны АЧХ. В области частоты раздела она близка к фильтрам 2 порядка, хотя на большом удалении возвращается к исходному для 1 порядка значению (6 дБ/октава).

      Введение шунтирующего резистора позволяет «приручить» горб на АЧХ, так что на кроссовер можно возложить и некоторые функции эквалайзера. Если шунт сделать на основе переменного резистора (или набора резисторов с переключателем), то можно проводить даже оперативную регулировку АЧХ в пределах 6-10 дБ. (рисунок 2):


Рисунок 2

      Однако фильтры первого порядка обеспечивают слишком малое затухание за пределами рабочей полосы, поэтому пригодны только при небольшой подводимой мощности или достаточно высокой частоте раздела (7-10 кГц). Поэтому в большинстве серьезных конструкций используют фильтры более высоких порядков, от второго до четвертого.
      Рассмотрим возможности воздействия на АЧХ для фильтров второго порядка, как самых распространенных. Для наглядности использована модель с большой индуктивностью. Те же результаты получаются и с традиционными пищалками, только параметры фильтров и степень воздействия на АЧХ будут другими. Для пищалок с малой индуктивностью шунт не обязателен.

      Первый способ — изменение добротности фильтра при неизменной частоте раздела за счет соотношения емкости и индуктивности фильтра (рисунок 3):


Рисунок 3

      Одновременное изменение емкости и индуктивности в кроссовере затруднено, поэтому данный метод для оперативной регулировки неудобен. Однако он незаменим в тех случаях, когда необходимая степень коррекции известна заранее, на этапе проектирования.

      Второй способ — регулировка добротности при помощи шунта (аналогично рассмотренному ранее способу для фильтра первого порядка). Исходная добротность разделительного фильтра при этом выбирается высокой (рисунок 4):


Рисунок 4

      Третий способ — введение резистора последовательно с пищалкой. Особенно удобен этот способ для пищалок индуктивностью свыше 100 мГн. В этом случае суммарный импеданс цепи «резистор-пищалка» в процессе регулирования изменяется незначительно, поэтому уровень сигнала практически не изменяется (рисунок 5):


Рисунок 5

      Выводы
      Стабилизирующие цепи не обязательны только для пищалок малой индуктивности (менее 0,05 мГн).
      Для пищалок с индуктивностью звуковой катушки 0,05-0,1 мГн наиболее выгодны параллельные стабилизирующие цепи (шунты).
      Для пищалок с индуктивностью звуковой катушки более 0,1 мГн можно использовать как параллельные, так и последовательные стабилизирующие цепи.
      Изменение сопротивления стабилизирующей цепи позволяет воздействовать на АЧХ.
      Для фильтров 1 порядка изменение параметров стабилизирующей цепи оказывает заметное влияние на частоту среза и параметры «горба». У фильтров 2 порядка частота среза определяется параметрами его элементов и зависит от индуктивности головки и параметров стабилизирующей цепи в меньшей степени.
      Величина резонансного «горба», вызванного индуктивностью пищалки, находится в прямой зависимости от сопротивления шунта и в обратной зависимости от сопротивления последовательного резистора.
      Величина резонансного «горба» в области частоты среза находится в прямой зависимости от добротности фильтра.
      Добротность фильтра пропорциональна результирующему сопротивлению нагрузки (ВЧ головки с учетом сопротивления стабилизирующей цепи).
            Фильтр повышенной добротности можно рассчитывать по стандартной методике, но на сниженное в 2-3 раза относительно номинального сопротивление нагрузки.

      Предложенные способы регулирования АЧХ применимы и к фильтрам более высоких порядков, но, поскольку число «степеней свободы» там возрастает, дать конкретные рекомендации в этом случае затруднительно. Пример изменения АЧХ фильтра третьего порядка за счет шунтирующего резистора приведен на рисунке 6:


Рисунок 6

      Видно, что АЧХ приобретает различный вид, что заметно влияет на тембр звучания. Кстати, лет 20 назад многие «домашние» трех-четырех полосные АС имели переключаемые АЧХ «normal/crystal/chirp» («гладкий-хрустальный-чирикающий»). Это достигалось изменением уровня полос СЧ и ВЧ.
      Переключаемые аттенюаторы используются в составе многих кроссоверов, причем по отношению к пищалке их можно рассматривать как комбинацию последовательных и параллельных стабилизирующих цепей. Воздействие их на результирующую АЧХ предсказать достаточно сложно, в этом случае удобнее прибегнуть к моделированию.


Рисунок 7

      На рисунке 7 приведена схема и АЧХ фильтра третьего порядка, разработанного автором для пищалок Prology RX-20s и EX-20s. В конструкции использованы конденсаторы К73-17 (2,2 мкФ, 63 В) и самодельные катушки индуктивности. Для снижения активного сопротивления они намотаны на ферритовых кольцах. Тип сердечника неизвестен: наружный диаметр 15 мм, магнитная проницаемость порядка 1000-2000. Поэтому подгонка индуктивности велась по прибору Ф-4320. Каждая катушка содержит 13 витков изолированного провода диаметром 1 мм.
      Качество звучания оказалось не в пример выше исходного, а регулирование АЧХ вполне соответствовало поставленной задаче. Однако следует отметить, что фильтр получился проблемным: входной импеданс имеет резко выраженный минимум, и возможно срабатывание защиты усилителя.


Адрес администрации сайта: [email protected]
   

НЕ НАШЕЛ, ЧТО ИСКАЛ? ПОГУГЛИ:

              СТРОКА ПОЛЬЗОВАТЕЛЬСКОГО ПОИСКА

Инженерный расчёт активных фильтров курсовая по технологии

Министерство образования и науки Украины Приазовский государственный технический Университет Кафедра АЭС и ЭП Пояснительная записка к курсовой работе «Инженерный расчёт активных фильтров» по курсу «Элементы автоматических устройств» Выполнил Принял Мариуполь 2003 Содержание Задание Краткие теоретические положения Расчёт фильтра Баттерворта нижних частот четвёртого порядка Расчёт фильтра Чебышева высоких частот четвёртого порядка с неравномерностью 0,5dB Расчёт полосового фильтра первого порядка Заграждающий фильтр первого порядка Выводы просто представить из двух асимптот: 1) [A]=0 Дб на нижних частотах f<<fg 2) На высоких частотах fg <<f [A]=1/ F 07 7RC -коэффициент усиления пропорционален частоте. При увеличении частоты в 10 раз коэффициент уменьшается в 10 раз, т.е -20 Дб на декаду. 3) Фильтр верхних частот Фильтр верхних частот -это схема, которая передает без изменений сигналы высоких частот, а на низких частотах обеспечивает затухание сигналов и опережение их по фазе относительно входных сигналов. Из формулы следует, что модуль комплексного коеф. усиления и угол сдвига определяются как Выражение для частоты среза совпадает с соответствующим выражением для фильтра нижних частот. Фазовый сдвиг на этой частоте составляет + 45. Как и для фильтра нижних частот, наиболее просто составить АЧХ в двойном логарифмическом масштабе с помощью асимптот: 1)|A|=1=`0 дБ, на высоких частотах f>>fg. 2) На низких частотах f<<fg |A|= F 07 7RC коэффициент усиления пропорционален частоте. Наклон асимптоты равняется + 20 дБ на декаду. 3) Активные фильтры Как и в пассивных фильтрах, RC-элементы являются неотъемлемой частью и в активных цепях. Меняя их параметры можно добиться нужного вида АЧХ. Если необходимо получить более быстрое уменьшение коэффициента передачи, можно включить n фильтров нижних частот последовательно. Передаточная функция фильтра нижних частот в общем виде выглядит так: , порядок фильтра определяется максимальной степенью P. Передаточная функция фильтра верхних частот в общем виде выглядит так: , порядок фильтра определяется максимальной степенью P. Заграждающий либо полосовой фильтр можно получить путём соединения ФВЧ и ФНЧ, при этом передаточная характеристика результирующего фильтра будет произведением передаточных функций полюсов. Расчёт фильтра Баттерворта нижних частот четвёртого порядка Диаграмма Боде исследуемого фильтра Реакция фильтра на сигнал прямоугольной формы 1. 2. 3. 4. 5. Расчёт фильтра Чебышева высоких частот четвёртого порядка с неравномерностью 0,5dB . 4. 5. Расчёт полосового фильтра первого порядка Моделирование работы схемы в пакете Multisim 2001 Исследуемая схема. Диаграмма Боде исследуемого фильтра Данный фильтр отвечает поставленными нами требованиям Импульсный режим работы фильтра 1. Диаграмма Боде исследуемого фильтра 1. 2. 3. 4. 5. Полученная модель заграждающего фильтра на основе аналитического расчёта вполне соответствует заданным характеристикам. Выводы по курсовому проекту В данной работе были спроектированы четыре типа активных фильтров, а так же составлены модели и проверены на работоспособность и соответствие поставленным требованиям. Проектирование активных аналоговых фильтров является довольно полезным опытом, так как фильтры находят применения во многих электронных устройствах, будь то средства связи, акустические системы или источники питания. Курсовой проект позволил не только получить опыт аналитического расчёта, но и научиться выбирать активные и пассивные элементы схем в соответствии с поставленными техническими требованиями и характеристиками.

Расчет lc фильтра для шим

LC – фильтры я оставил на десерт, подобно бутылке благородного вина, покрытой слоем вековой пыли. Это антиквариат, который на Сотбисе не купишь!

Как ни крути, а не получил бы Александр Степаныч наш Попов звание почётного инженера-электрика, не направь он искровой разряд напрямик в колебательный контур для обретения благословения свыше и резонанса с передающей антенной.
И заскучала бы братва копателей свободной энергии эфира, не изобрети Никола Тесла свой резонансный трансформатор и электрический автомобиль с неведомой коробочкой. А то и вовсе, заширялась бы в подъездах, лишённая идей вселенского масштаба.

И начнём мы с расчёта самого простого LC-фильтра – колебательного контура.

Включённый по приведённой на рис.1 схеме, он представляет собой узкополосный полосовой фильтр, настроенный на частоту fо= 1/2π√ LС .
На резонансной частоте сопротивление контура равно:
Rо = pQ, где р – характеристическое сопротивление, равное реактивному сопротивлению катушки и конденсатора.
Оно в свою очередь рассчитывается по формуле р = √ L/C .

На низких (звуковых) частотах конденсаторы практически не вносят потерь, поэтому добротность контура равна добротности катушки индуктивности, величина которой напрямую зависит от активного сопротивления катушки. Чем ниже частота, тем больше витков и тоньше провод, тем проще его измерить тестером. Если эта попытка удалась, то Q=2πfL/R, где R – активное сопротивление катушки индуктивности.
На радиочастотах значение активного сопротивления катушки может составлять доли ома, поэтому для расчёта добротности надо – либо найти сопротивление в Омах по формуле R= 4ρ*L/(πd²), где ρ — удельное сопротивление меди, равное 0,017 Ом•мм²/м, L – длина в метрах, d – диаметр провода в мм, либо вооружиться генератором сигналов, каким-либо измерителем уровня выходного сигнала с высоким внутренним сопротивлением, и определить добротность экспериментально.
К тому же на высоких частотах возможно проявление влияния добротности конденсатора, особенно если он окажется варикапом, хотя современные недорогие керамические изделия (например, фирмы Murata) имеют значение параметра добротности – не менее 800.

Нарисуем табличку с расчётом фильтра для низкочастотных приложений.

ТАБЛИЦА ДЛЯ LC- РЕЗОНАНСНОГО (ПОЛОСОВОГО) ФИЛЬТРА ДЛЯ НЧ.

Если параметр активного сопротивления катушки R опущен, его значение принимается равным 200 омам.
Необходимо отметить, что все полученные в таблице данные верны и для последовательного колебательного контура. При этом, если мы хотим использовать свойства контура полностью, т. е. получить острую резонансную кривую, соответствующую конструктивной добротности, то параллельный контур надо нагружать слабо, выбирая R1 и Rн намного больше Rо (на практике десятки кОм), для последовательного же контура, сопротивление генератора R1 наоборот должно быть на порядок меньше характеристического сопротивления ρ.

Теперь, нарисуем таблицу для расчёта высокочастотных резонансных контуров.
Тут на добротность влияет не только активное сопротивление катушек, но и другие факторы, такие как – потери в ферритах, наличие экрана, эффект близости витков и т. д. Поэтому вводить этот параметр в качестве входного я не стану – будем считать, что добротность катушки вы измерили, или подсмотрели в документации на готовые катушки. Естественным образом значение добротности катушки должно измеряться на резонансной частоте контура, ввиду прямой зависимости этой величины от рабочей частоты (Q=2πfL/R).
К тому же я добавлю сюда параметр добротности конденсатора, особенно актуальный в случае применения варикапов.
По умолчанию (для желающих оставить эти параметры без внимания), добротность катушки примем равной 100, конденсатора – 1000, а для испытывающих стремление измерить эти параметры в радиолюбительских условиях, рекомендую посетить страницу ссылка на страницу .

ТАБЛИЦА ДЛЯ LC- РЕЗОНАНСНОГО (ПОЛОСОВОГО) ФИЛЬТРА ДЛЯ ВЧ.

Теперь плавно переходим к LC фильтрам верхних и нижних частот (ФВЧ и ФНЧ).

Рис.2

Крутизна спада АЧХ этих фильтров в полосе подавления – 12 дБ/октаву, коэффициент передачи в полосе пропускания К=1 при R1 << ρ << Rн, где R1 – внутреннее сопротивление генератора, Rн – сопротивление нагрузки, а ρ – характеристическое сопротивление фильтра.
Однако наилучшие параметры, с точки зрения равномерности АЧХ и передачи максимальной мощности в нагрузку, обеспечиваются при R1=Rн=ρ. В этом случае фильтр является согласованным, правда коэффициент передачи в полосе пропускания становится равным К=0.5.
Ну да ладно, ближе к делу.

ТАБЛИЦА LC- ФИЛЬТРОВ ВЕРХНИХ и НИЖНИХ ЧАСТОТ.

А если надо рассчитать L и C при известных значениях Fср и ρ ? Не вопрос,

ТАБЛИЦА РАСЧЁТА ЭЛЕМЕНТОВ LC- ФИЛЬТРОВ ВЕРХНИХ и НИЖНИХ ЧАСТОТ.

Данные ФВЧ и ФНЧ называются Г-образными.
Для получения более крутых скатов АЧХ используют два или более Г-образных звеньев, соединяя их последовательно, чтобы образовать Т-образное звено (на Рис.3 сверху), или П-образное звено (на Рис.3 снизу). При этом получаются ФНЧ третьего порядка. Обычно, ввиду меньшего количества катушек, предпочитают П-образные звенья.

Рис.3

ФВЧ конструируют подобным же образом, лишь катушки заменяются конденсаторами, а конденсаторы – катушками.

Широкополосные полосовые LC – фильтры получают каскадным соединением ФНЧ и ФВЧ.

Что касается многозвенных LC-фильтров высоких порядков, то более грамотным решением (по сравнению с последовательным соединением фильтров низших порядков) будет построение подобных устройств с использованием полиномов товарищей Чебышева или Баттерворта.

Именно такие фильтры 3-го, 5-го и 7-го порядков мы и рассмотрим на следующей странице.

LC – фильтры я оставил на десерт, подобно бутылке благородного вина, покрытой слоем вековой пыли. Это антиквариат, который на Сотбисе не купишь!

Как ни крути, а не получил бы Александр Степаныч наш Попов звание почётного инженера-электрика, не направь он искровой разряд напрямик в колебательный контур для обретения благословения свыше и резонанса с передающей антенной.
И заскучала бы братва копателей свободной энергии эфира, не изобрети Никола Тесла свой резонансный трансформатор и электрический автомобиль с неведомой коробочкой. А то и вовсе, заширялась бы в подъездах, лишённая идей вселенского масштаба.

И начнём мы с расчёта самого простого LC-фильтра – колебательного контура.

Включённый по приведённой на рис.1 схеме, он представляет собой узкополосный полосовой фильтр, настроенный на частоту fо= 1/2π√ LС .
На резонансной частоте сопротивление контура равно:
Rо = pQ, где р – характеристическое сопротивление, равное реактивному сопротивлению катушки и конденсатора.
Оно в свою очередь рассчитывается по формуле р = √ L/C .

На низких (звуковых) частотах конденсаторы практически не вносят потерь, поэтому добротность контура равна добротности катушки индуктивности, величина которой напрямую зависит от активного сопротивления катушки. Чем ниже частота, тем больше витков и тоньше провод, тем проще его измерить тестером. Если эта попытка удалась, то Q=2πfL/R, где R – активное сопротивление катушки индуктивности.
На радиочастотах значение активного сопротивления катушки может составлять доли ома, поэтому для расчёта добротности надо – либо найти сопротивление в Омах по формуле R= 4ρ*L/(πd²), где ρ — удельное сопротивление меди, равное 0,017 Ом•мм²/м, L – длина в метрах, d – диаметр провода в мм, либо вооружиться генератором сигналов, каким-либо измерителем уровня выходного сигнала с высоким внутренним сопротивлением, и определить добротность экспериментально.
К тому же на высоких частотах возможно проявление влияния добротности конденсатора, особенно если он окажется варикапом, хотя современные недорогие керамические изделия (например, фирмы Murata) имеют значение параметра добротности – не менее 800.

Нарисуем табличку с расчётом фильтра для низкочастотных приложений.

ТАБЛИЦА ДЛЯ LC- РЕЗОНАНСНОГО (ПОЛОСОВОГО) ФИЛЬТРА ДЛЯ НЧ.

Если параметр активного сопротивления катушки R опущен, его значение принимается равным 200 омам.
Необходимо отметить, что все полученные в таблице данные верны и для последовательного колебательного контура. При этом, если мы хотим использовать свойства контура полностью, т. е. получить острую резонансную кривую, соответствующую конструктивной добротности, то параллельный контур надо нагружать слабо, выбирая R1 и Rн намного больше Rо (на практике десятки кОм), для последовательного же контура, сопротивление генератора R1 наоборот должно быть на порядок меньше характеристического сопротивления ρ.

Теперь, нарисуем таблицу для расчёта высокочастотных резонансных контуров.
Тут на добротность влияет не только активное сопротивление катушек, но и другие факторы, такие как – потери в ферритах, наличие экрана, эффект близости витков и т. д. Поэтому вводить этот параметр в качестве входного я не стану – будем считать, что добротность катушки вы измерили, или подсмотрели в документации на готовые катушки. Естественным образом значение добротности катушки должно измеряться на резонансной частоте контура, ввиду прямой зависимости этой величины от рабочей частоты (Q=2πfL/R).
К тому же я добавлю сюда параметр добротности конденсатора, особенно актуальный в случае применения варикапов.
По умолчанию (для желающих оставить эти параметры без внимания), добротность катушки примем равной 100, конденсатора – 1000, а для испытывающих стремление измерить эти параметры в радиолюбительских условиях, рекомендую посетить страницу ссылка на страницу .

ТАБЛИЦА ДЛЯ LC- РЕЗОНАНСНОГО (ПОЛОСОВОГО) ФИЛЬТРА ДЛЯ ВЧ.

Теперь плавно переходим к LC фильтрам верхних и нижних частот (ФВЧ и ФНЧ).

Рис.2

Крутизна спада АЧХ этих фильтров в полосе подавления – 12 дБ/октаву, коэффициент передачи в полосе пропускания К=1 при R1 << ρ << Rн, где R1 – внутреннее сопротивление генератора, Rн – сопротивление нагрузки, а ρ – характеристическое сопротивление фильтра.
Однако наилучшие параметры, с точки зрения равномерности АЧХ и передачи максимальной мощности в нагрузку, обеспечиваются при R1=Rн=ρ. В этом случае фильтр является согласованным, правда коэффициент передачи в полосе пропускания становится равным К=0.5.
Ну да ладно, ближе к делу.

ТАБЛИЦА LC- ФИЛЬТРОВ ВЕРХНИХ и НИЖНИХ ЧАСТОТ.

А если надо рассчитать L и C при известных значениях Fср и ρ ? Не вопрос,

ТАБЛИЦА РАСЧЁТА ЭЛЕМЕНТОВ LC- ФИЛЬТРОВ ВЕРХНИХ и НИЖНИХ ЧАСТОТ.

Данные ФВЧ и ФНЧ называются Г-образными.
Для получения более крутых скатов АЧХ используют два или более Г-образных звеньев, соединяя их последовательно, чтобы образовать Т-образное звено (на Рис.3 сверху), или П-образное звено (на Рис.3 снизу). При этом получаются ФНЧ третьего порядка. Обычно, ввиду меньшего количества катушек, предпочитают П-образные звенья.

Рис.3

ФВЧ конструируют подобным же образом, лишь катушки заменяются конденсаторами, а конденсаторы – катушками.

Широкополосные полосовые LC – фильтры получают каскадным соединением ФНЧ и ФВЧ.

Что касается многозвенных LC-фильтров высоких порядков, то более грамотным решением (по сравнению с последовательным соединением фильтров низших порядков) будет построение подобных устройств с использованием полиномов товарищей Чебышева или Баттерворта.

Именно такие фильтры 3-го, 5-го и 7-го порядков мы и рассмотрим на следующей странице.

В статье речь пойдёт про расчёт простейших фильтрующих цепей для сглаживания широтно-импульсной модуляции. Что такое ШИМ, где он применяется и как его реализовать читайте в отдельной статье.

Первое, на чём следует заострить внимание – это назначение цепи, для которой вы собрались строить фильтр. Немного упрощая схемы с ШИМ можно поделить на два типа:

  • Сигнальные цепи с ШИМ
  • Силовые цепи с ШИМ

Примером сигнального ШИМ служит, например, простейший ЦАП, под силовым ШИМ чаще всего имеется ввиду ШИМ-сигнал на выходе силовых ключей, например в импульсных источниках питания (ИИП). Строго говоря, в источниках питания сам сигнал ШИМ тоже используется в сигнальной цепи (управление транзисторами) и на выходе таких источников сигнал повторяет форму управляющих сигналов, однако имеет более высокую мощность, потому они требуют фильтров позволяющих пропускать большие мощности.

Фильтрация ШИМ в сигнальных цепях


Рис.1. Простейший фильтр нижних частот – интегрирующая RC-цепь и её АЧХ.

Основная характеристика фильтра это частота среза (на рисунке 1 обозначена угловая частота среза – ωс) – амплитуда колебаний данной данной частоты на выходе фильтра ослабляется до уровня

0.707 (-3 Дб) от входного значения. Частота среза определяется по следующей формуле:

Тут R и С – сопротивление резистора в омах и ёмкость конденсатора в фарадах. Необходимо помнить, что для корректной работы сглаживающего фильтра постоянная времени RC-цепочки (τ = R · C) должна быть как можно меньше периода ШИМа, тогда за один период не будет происходить полный заряд-разряд конденсатора.

Следующий важный параметр, позволяющий расчитать ослабление колебаний на заданной частоте это коэффициент передачи фильтра – это отношение K = Uвых/Uвх. Для данной RC-цепочки коэффициент передачи рассчитывается следующим образом:

Зная эти формулы и учтя постоянное падение напряжения на резисторе можно приближённо рассчитать фильтр с нужными характеристиками – например, задавшись имеющейся ёмкостью, либо необходимым уровнем пульсаций.

Калькулятор ШИМ-фильтра на RC-цепочке

Входные данные:Расчётные значения:
Частота ШИМГц
Коэффициент заполнения ШИМ
(duty cycle)
%
Напряжение на входеВ
Сопротивление резистора фильтраОм
Сопротивление нагрузки RнОм
Ёмкость конденсаторамкФ
Частота среза RC-фильтра– ГцАмплитуда пульсаций на частоте ШИМ– ВКоэффициент пульсаций на выходе– %Напряжение на выходе
(с учётом потерь на резисторе фильтра)– В

Постоянная времени RC-цепи

Постоянная времени цепи (R · C) не должна быть сильно меньше периода ШИМ!

Потери на резисторе фильтра
(не менее)– Вт

Обратите внимание – если вы хотите получать из ШИМ-сигнала сглаженный синусоидальный сигнал, необходимо чтобы частота среза фильтра была выше максимальной частоты сигнала, а значит частота ШИМ должна быть ещё выше.

Фильтрация ШИМ в силовых цепях


Рис.2. Фильтр нижних частот на LC-контуре и его АЧХ.

LC-фильтр представляет из себя элементарный колебательный контур, который имеет собственную частоту резонанса, поэтому его реальная АЧХ будет несколько отличаться от АЧХ, приведённой на рисунке 2.

Поскольку речь в данной статье идёт о фильтре для силовых цепей, при расчёте фильтра нужно учитывать, что основная гармоника входящего напряжения тоже должна ослабляться фильтром, следовательно, его резонансная частота должна быть ниже частоты ШИМ.

Формула для расчёта частоты резонанса LC-контура:

f = 1/(2 · π · (L · C) 0.5 )

Если частота резонанса контура совпадёт с частотой ШИМ, LC-контур может перейти в режим генерации, тогда на выходе может случиться конфуз, посему предлагаю вам данного недоразумения тщательно избегать. Кроме того, при проектировании данного фильтра есть ещё несколько нюансов, которые неплохо бы соблюдать для получения желаемого результата, а именно:

    Для исключения резонансных явлений на одной из высокочастотных гармонических составляющих ёмкость конденсатора желательно находить из условия равенства волнового сопротивления фильтра сопротивлению нагрузки:
  • Для сглаживания пульсаций таким фильтром желательно, чтобы ёмкостное сопротивление конденсатора для низшей частоты пульсации было как можно меньше сопротивления нагрузки, а также много меньше индуктивного сопротивления дросселя для первой гармоники.
  • Комплексный коэффициент передачи LC-фильтра рассчитывается по следующей формуле:

    где n – номер гармонической составляющей входного сигнала, i – мнимая единица, ω = 2πf, L – индуктивность дросселя (Гн), C – ёмкость конденсатора (Ф), R – сопротивление нагрузки (Ом).

    Из формулы очевидно, что чем выше гармоника, тем лучше она подавляется фильтром, следовательно, достаточно рассчитывать уровень только для первой гармоники.

    Чтобы перейти от комплексного представления коэффициента передачи к показательному, нужно найти модуль комплексного числа. Для тех, кто (как и я) спал на парах матана в институте, напомню, модуль комплексного числа считается очень просто:

    r = |Z| = (x 2 + y 2 ) 0.5

    Так как у нас в формуле коэффициента дробь, просто так сходу посчитать модуль не получится и проще всего посчитать это всё, например в MathCad’е. А для тех, кому лень делать всё самим, я запилил весь расчёт в этот прекрасный калькулятор. Пользуйтесь:

    Калькулятор силового ШИМ-фильтра на LC-контуре

    Частота ШИМГц
    Коэффициент заполнения ШИМ
    (duty cycle)
    %
    Напряжение на входеВ
    Сопротивление нагрузкиОм
    Индуктивность катушкимГн
    Ёмкость конденсаторамкФ
    Амплитуда пульсаций на частоте ШИМВ
    Резонансная частота контураГц
    Напряжение на выходе
    (потери не учитываются)
    В

    Обратите внимание – при использовании LC-фильтра следует помнить, что из-за наличия в цепи индуктивности, на выходе могут появляться выбросы обратной полярности. Если полярность импульсов на входе не изменяется (например для изменения направления вращения двигателя) для ограничения амлитуды отрицательных выбросов параллельно конденсатору (?) можно включать диод Шоттки.

    Расчет фильтра нижних частот

    Название: Расчет фильтра нижних частот

    Вид работы: курсовая работа

    Рубрика: Коммуникации и связь

    Размер файла: 777,63 Kb

    Скачать файл: referat.me-170615.docx

    Краткое описание работы: Содержание Введение 1 Физические основы и принцип действия широкополосных фильтров 2 Пример расчета фильтра нижних частот на заданные параметры Заключение

    Содержание

    Введение

    1 Физические основы и принцип действия широкополосных фильтров

    2 Пример расчета фильтра нижних частот на заданные параметры

    Заключение

    Список использованной литературы

    Введение

    Во многих радиотехнических устройствах часто возникает необходимость выделения заданных частотных диапазонов из имеющегося спектра частот. Выделение требуемой полосы частот с очень малым затуханием осуществляется фильтрами.

    В начале нашего столетия электрические фильтры, составленные из ряда катушек индуктивности и конденсаторов, получили широкое применение в технике. Благодаря их применению оказалось возможным осуществление многих магистралей дальней телефонной, телеграфной и других видов связи.

    Известный интерес представляют пассивные цепочные, или лестничные фильтры, состоящие из комбинаций элементов L и C и не требующие источника питания. Пассивные фильтры могут обладать как широкими полосами пропускания, так и очень узкими.

    При расчете фильтра, с одной стороны, необходимо определить, с какими допустимыми искажениями передается входной сигнал, являющийся функцией частоты или времени, на выход фильтра, и, с другой стороны, из каких конкретных элементов должен состоять этот фильтр. Получение наивыгоднейших выходных характеристик с минимальными искажениями и создание принципиальной схемы фильтра с минимальным числом элементов, осуществляющей требуемую передачу сигнала, является содержанием расчета фильтров.

    1 Физические основы и принцип действия широкополосных фильтров

    В электрических, радиотехнических и телемеханических установках часто решается задача: из совокупного сигнала, занимающего широкую полосу частот, выделить один или несколько составляющих сигналов с более узкой полосой. Сигналы заданной полосы выделяют при помощи частотных электрических фильтров.

    К частотным электрическим фильтрам различной аппаратуры предъявляются разные, порой противоречивые требования. В одной области частот, которая называется полосой пропускания, сигналы не должны ослабляться, а в другой, называемой полосой задерживания, ослабление сигналов не должно быть меньше определенного значения. Фильтр считают идеальным, если в полосе пропускания отсутствует ослабление сигналов и фазо-частотная характеристика линейна (нет искажения формы сигналов), а вне полосы пропускания сигналы на выходе фильтра отсутствуют.

    Фильтры могут быть однозвенные (первого порядка), двухзвенные (второго порядка) и многозвенные (n- го). Чем выше порядок фильтра, тем круче его амплитудно-частотная характеристика и тем более она похожа на его идеальную характеристику. Фильтр любого порядка можно построить путем каскадного соединения фильтров первого и второго порядков.

    Электрический фильтр представляет собой четырехполюсник, предназначенный для выделения из состава сложного электрического колебания частотных составляющих, расположенных в заданной полосе частот, и подавления тех составляющих, которые расположены в других полосах частот. Первая из названных полос представляет собой полосу пропускания, а вторая – полосу задерживания.

    По взаимному расположению полос пропускания и полос задерживания различают фильтры нижних частот, фильтры верхних частот, полосовые и режекторные фильтры.

    Фильтры нижних частот (ФНЧ) пропускают сигналы частот от 0 до и задерживающей колебания любых белее высоких частот (рис.1).

    LC–фильтр нижних частот (рис. 6.3,а) пропускает электрические колебания в полосе частот от 0 до

    .

    Это объясняется тем, что на низких частотах сопротивление индуктивного элемента XL фильтра мало, а емкостного XC – велико и электрические колебания проходят со входа на выход почти без ослабления. С увеличением частоты сопротивление индуктивного элемента возрастает, а емкостного – снижается и коэффициент передачи фильтра уменьшается (рисунок 1–б).

    Фильтры верхних частот (ФВЧ), напротив, пропускают сигналы, частоты которых простираются от до , и задерживают сигналы более низких частот(рис.2).

    Полосовые фильтры (ПФ) пропускают сигналы в полосе частот от до (рис.3).

    Режекторные (заграждающие) фильтры (РФ) задерживают сигналы в полосе частот от до (рис.4).

    (а)

    (б)

    Рисунок — 1 Фильтры нижних частот (а) и его АЧХ (б).

    Рисунок — 2 Фильтры верхних частот

    Рисунок – 3 Полосовые фильтры

    Рисунок – 4 Заграждающие фильтры

    Широкополосный фильтр состоит из звеньев, включающих в себя только элементарные контуры. Поэтому механизм фильтрации в широкополосных фильтрах определяется в основном процессами, происходящими в элементарны контурах, т. е. в двухэлементных двухполюсниках, имеющих одну резонансную частоту.

    Полное сопротивление Z любого элементарного контура состоит из активной R и реактивной Xсоставляющих:

    Z = R + jX(1)

    или

    Y = G + jB(2)

    где Y – полная проводимость

    G – активная составляющая полной проводимости

    B – реактивная составляющая поной проводимости

    Реактивное сопротивление последовательного контура на частоте = равно нулю, активное сопротивление при этом мало, так как оно определяется только потерями в контуре; полное сопротивление контура близко к нулю. Поэтому последовательные контуры применяются в параллельных плечах Т-образных звеньев фильтров верхних частот (рис.5)., так как они оказывают сильное шунтирующее действие на частотах, близких к резонансной.

    Рисунок – 5 Схема звена ФВЧ и резонансные характеристики последовательного контура

    Реактивная проводимость = равна нулю. Следовательно, такой контур является фильтром-пробкой для частот, близких к резонансной. Поэтому параллельные контуры применяются в последовательных плечах П-образных звеньев фильтров нижних частот (рис.6). Резонансные частоты элементарных контуров равны частотам бесконечного затухания соответствующих звеньев =. Резонансные частоты контуров, или частоты бесконечного затухания, для ФНЧ располагаются выше частот полосы пропускания ( >), а для ФВЧ – ниже (<). Параллельные плечи фильтра шунтируют частоты полосы задерживания в то время, как последовательные являются для них пробкой. В полосе пропускания сопротивление параллельных плеч фильтра , напротив, велико, а сопротивление последовательных плеч близко к нулю.

    Рисунок – 6 Схема звена ФНЧ и резонансные характеристики параллельного контура

    Благодаря этому через фильтр беспрепятственно проходят частоты полосы пропускания. В этой связи становится понятным, почему в широкополосных фильтрах все звенья настраиваются на частоты полосы задерживания.

    Крутизна частотной характеристики фильтра в переходной области зависит от числа звеньев. Широкополосный фильтр всегда состоит из фильтра нижних частот и фильтра верхних частот. Полоса пропускания широкополосный фильтра — образуется благодаря перекрытию полос пропускания ФНЧ (0 -) и ФИЧ (- ) — (рис.7)

    Рисунок – 7 Образование полосы пропускания широкополосного фильтра

    2 Пример расчета фильтра нижних частот на заданные параметры

    Аналитический метод расчета цепочных фильтров основан на нахождении оптимальных параметров фильтра по заданной характеристике собственного или рабочего затухания. При этом реализуется фильтр, удовлетворяющий заданным условиям, при минимальном числе элементов, что гарантирует минимальное искажение в полосе пропускания. Отметим, что, так как последовательно и параллельно-производные звенья фильтров являются дуальными, т.е. взаимообратными, а свойства таких схем в отношении передачи энергии аналогичны, то количество расчетных формул, используемых для расчета фильтров, будет вдвое уменьшено. Поэтому ведется единый расчет фильтра.

    Требуется рассчитать фильтр, удовлетворяющий следующим техническим требованиям:

    1. Полоса пропускания

    = 50 кГц, = 100 кГц,

    где и соответственно нижняя и верхняя граничные полосы пропускания.

    2. Полоса задерживания

    От = 115 кГц и выше

    где и соответственно верхняя и нижняя граничные полосы задерживания.

    3. Рабочее затухание в полосе пропускания

    7дБ = =0,805 неп

    4. Рабочее затухание в полосе задерживания

    50 дБ = =5,75 неп

    5.Фильтр включается между сопротивлением нагрузок

    = 1000 Ом

    6.Отклонение характеристического сопротивления от номинального в полосе пропускания (коэффициент несогласованности)

    = 10 % =0,1

    7. Амплитуда входного сигнала

    = 5 В

    9. Коэффициент использования полосы пропускания.

    8. Рекомендуется выполнить катушки индуктивностей на альсиферовых сердечниках.

    Прежде чем приступить к расчету фильтра, необходимо определить критерий полосности

    =

    Так как n < 2, рассчитываем фильтр как полосовой.

    Зная вычислим ,затем определим затухание несогласованности

    Выбрав класс фильтра по сопротивлению (обычно 2-ой), т.е. =2, найти параметр

    Далее определить расчетный параметр полосы пропускания

    и затем расчетный параметр

    Найдем коэффициент использования полосы пропускания

    И только потом можно определить коэффициент использования полосы задерживания и теоретическую частоту среза. Для реальных фильтров . Заданные параметры и = 100 кГц, определим теоретическую частоту среза

    Так как теоретическая частота среза лежит в пределах переходной области, она удовлетворяет предъявленным условиям. Исходя из этого, определяем предварительный коэффициент использования полосы пропускания

    Поскольку величина не превышает предельно достижимого в реальных фильтрах значения 0,98, останавливаемся на этих значениях и . Определяем коэффициент использования полосы задерживания и расчетные параметры — расчетный параметр и — расчетный параметр полосы задерживания:

    Зная , определяем затухание одного звена фильтра с учетом потерь в полосе задерживания

    Далее находим минимальное собственное затухание всего фильтра в полосе задерживания, которое необходимо для обеспечения гарантируемого минимума затухания

    По найденным значениям и определяем число звеньев N (или, что то же самое, класс фильтра по затуханию)

    Для выбранного округленного числа звеньев N=4 пересчитываем величины , , , а также . Уточнив все эти параметры проверяем отклонение характеристического сопротивления от номинального.

    Зная , пересчитываем коэффициент использования полосы задерживания

    ,

    используя который, определяем уточненную верхнюю теоретическую частоту среза

    Полученное значение теоретической частоты среза удовлетворяет исходным требованиям, так как находится в переходной области. Далее определяем уточненный коэффициент использования полосы пропускания

    ,

    Теперь по сути дела возвращаемся в начало расчета. Для проверки отклонения характеристического сопротивления фильтра от номинального сопротивления нагрузки в полосе пропускания рассчитываем параметры , и (по данным полосы пропускания):

    Выбираем заранее фильтр 2-го класса по сопротивлению и определяем затухание несогласованности в полосе пропускания

    и затухание эха

    После определяем величину отклонения характеристического сопротивления от номинального в полосе пропускания

    Что удовлетворяет исходным техническим требованиям. Таким образом, на основании проведенного расчета выбран фильтр 2-го класса по сопротивлению с числом звеньев N=4. Отклонение характеристического сопротивления от номинального в полосе пропускания составляет 3,558 %, коэффициент использования полосы пропускания =0,885, теоретическая частота среза =113,016 кгц.

    Расчетные параметры необходимые в дальнейшем,

    =0,43 =0,4659=113,016 кгц.

    Так как фильтр состоит из четырех звеньев, то для каждого звена рассчитываем коэффициент и соответственно расчетный параметр m.

    где

    =1,2,3,4 — № звена

    N= 4 – число звена

    Таким образом

    Проверкой правильности расчета при четном числе звеньев служит соблюдение следующих соотношений:

    , и т.д.

    т.е. в рассматриваемом примере и т.д.

    Для расчета коэффициентов m полагаем, что =0,43, тогда

    Затем определяем

    Для получения наилучшего согласования фильтра с нагрузкой начинаем и оканчиваем фильтр полузвеньями, у которых значение коэффициента m ближе к . В рассматриваемом примере этому требованию удовлетворяет коэффициент .

    Внутри фильтра звенья с различными коэффициентами могут соединяться в произвольном порядке, но при условии соблюдения согласования.

    Затем приступаем к выбору конкретной схемы фильтра нижних частот. При этом нужно стремиться к выбору звеньев с наименьшим числом катушек индуктивности, поскольку именно эти элементы наиболее усложняют производство и настройку фильтра. Так как класс фильтра определяется классом конечных полузвеньев, то для них выбираем звено типа 2А1н (звено Т-образного вида, 2-го класса по сопротивлению, с одной стороной среза фильтра нижних частот) и делим его пополам, для того, чтобы не увеличивать общее число звеньев. При этом получаются оконечные полузвенья Г-образного вида. В качестве промежуточных звеньев используем звенья типа 1В1н (звено П-образного вида, первого класса по сопротивлению, с одной частотой среза, фильтра нижних частот). Далее составляем полную принципиальную схему всего фильтра нижних частот рисунок 8.

    звена 2А1н 1В1н 1В1н1В1нзвена 2А1н

    Рисунок 8 – Полная принципиальная схема фильтра нижних частот

    Первый этап расчета заканчиваем вычислением частот минимального затухания, лежащих в полосе задерживания.

    соответственно

    Формула для расчета частот минимального затухания

    Аналогично формуле для расчета с той лишь разницей, что вместо параметра подставляется . Коэффициент , аналогичный коэффициенту , рассчитывается по формуле

    где=1,2,3,4 – № звена

    N= 4 – число звеньев

    =0,3986 – ранее определенный параметр

    Таким образом

    Проверка правильности расчета коэффициентов :

    , и т.д.

    где

    т.е. в рассматриваемом примере и т.д.

    Далее для каждого звена определяем параметр и частоту минимального затухания:

    Частота последнего минимума затухания должна совпадать с верхней граничной частотой полосы задерживания. Кроме того должно соблюдаться строгое чередование частот бесконечного и минимального затухания. Правильность расчета подтверждается в рассматриваемом примере равенством и таблицей 1.

    Таблица 1

    Заметим, что наибольшему значению m соответствует наиболее удаленная от полосы пропускания частота бесконечного затухания, а наименьшему значению m соответствует ближайшая к переходной области частота бесконечного затухания. Все частоты и находятся в полосе задерживания. Не допускается расположение частот и в полосе пропускания.

    Определив частоты бесконечного и минимального затухания, а так же значения коэффициентов для всех звеньев и составив полную принципиальную схему фильтра нижних частот (ФНЧ состоит из трех звеньев типа 1В1н и двух полузвеньев типа 2А1н), приступаем к расчету номинальных значений элементы схемы ФНЧ. Начинаем с определения величины расчетного сопротивления , а затем единичной индуктивности и единичной емкости .

    Коэффициент нагрузки рассчитывается графоаналитическим способом. Суть расчета сводится к тому, что выбирается такое соотношение между сопротивлением нагрузки и номинальным характеристическим сопротивлением, при котором в заданной полосе частот обеспечивается наилучшее согласование. Расчет характеристического сопротивления ведется только для оконечного полузвена, ибо если выполняются условия согласования характеристического сопротивления с сопротивлением нагрузки для оконечного полузвена на всех частотах полосы пропускания, то они выполняются и для всех промежуточных звеньев (данные расчета приведены в таблице 2).

    Таблица 2

    Оконечное полузвено Ом =113,016 кгц.
    f, кгц
    50 0,4424 0,8968 0,87128 1,02929 1029,29 1,03592
    55 0,4867 0,8736 0,84421 1,0348 1034,8 1,0415
    60 0,5309 0,8474 0,8146 1,04027 1040,27 1,04697
    65 0,5751 0,81805 0,7825 1,0454 1045,4 1,0521
    70 0,6194 0,7851 0,7477 1,0500 1050,0 1,0568
    75 0,6636 0,7481 0,71037 1,05311 1053,11 1,053865
    80 0,7079 0,7064 0,6704 1,05369 1053,69 1,0605
    85 0,7521 0,6590 0,62797 1,0494 1049,4 1,0562
    90 0,7964 0,6048 0,58285 1,03766 1037,66 1,0443
    95 0,8406 0,5417 0,53526 1,01203 1012,03 1,01855
    100 0,8848 0,4659 0,48511 0,9604 960,4 0,9666

    Коэффициент согласования – это отношение характеристического сопротивления к сопротивлению нагрузки, т.е. . Чем меньше коэффициент согласования отличается от единицы, тем лучше согласованы характеристическое сопротивление и сопротивление нагрузки. Графоаналитический метод заключается в симметрировании кривой относительно значения (т.е. уравнивания наибольшего и наименьшего значений в полосе пропускания).

    Исходя из этого, определяется коэффициент нагрузки , позволяющий затем вычислить как оптимальное номинальное характеристическое сопротивление. Проводя симметрирование, перемещаем кривую вдоль оси ординат относительно значения (в рассматриваемом примере вниз) так, чтобы максимальное положительное и максимальное отрицательное отклонения функции от значения были приблизительно равны.

    Рисунок 9 – График симметрирования характеристического сопротивления ФНЧ

    В рассматриваемом примере экспериментальное значение функции соответствует значениям и . На частоте коэффициент согласования . На частоте коэффициент согласования .

    Из этого обобщенного графика можно сделать заключение о форме характеристики затухания фильтра нижних частот. Положительное отклонение от значения свидетельствует об обеспечении неискаженной передачи частот полосы пропускания в диапазоне частот и ; отрицательное отклонение свидетельствует о наличие искажений для некоторых частот полосы пропускания (наблюдается так называемый завал фронтов амплитудно-частотной характеристики).

    Максимально положительное и максимально отрицательное отклонение на этих частотах составляют соответственно ()

    Разность отклонений

    Среднее отклонение

    Новое значение коэффициента согласования на частоте с учетом среднего отклонения

    Коэффициент несогласованности и коэффициент нагрузки на частоте соответственно равны

    Полагая коэффициент нагрузки на частотах и одинаковым, находим коэффициент согласования и коэффициент несогласованности на частоте :

    Так как значения и ничтожно мало отличаются друг от друга, коэффициент нагрузки выбираем окончательно .

    Определяем расчетное характеристическое сопротивление

    Ом

    Следует отметить, что, так как и мало отличается от , можно было бы пренебречь несогласованностью при расчете и полагать Ом.

    Вычисляем величины единичной индуктивности и единичной емкости исходного звена типа .

    , ,

    Рисунок 10 – Расчетные соотношения для П-образного звена типа 1В1н фильтра нижних частот.

    Учитывая составленную полную принципиальную схему фильтра нижних частот (см. рисунок 8), рассчитываем номинальные величины элементов каждого звена схемы. Звено типа 1В1н рассчитывается следующим образом:

    Зная коэффициенты и можно рассчитать элементы всех звеньев (рисунок 10), учитывая при этом, что для каждого звена требуются свои значения m, а значения и постоянны для всех звеньев. Расчет дается для одного типового промежуточного звена и оконечного полузвена. Значения элементов остальных звеньев сведены в таблице 3.

    Расчет промежуточного полузвена

    ,,,

    ,

    ,

    ,

    Сразу же проверяем резонансную частоту контура, значение которой должно совпадать со значением частоты бесконечного затухания при том же значении m.

    ()

    Таблица 3

    звена

    Расчетный

    коэффициент

    ,

    кГц

    0,31642 1,4099 0,63284 0,8974 1973,8 442,97 119,65
    0,19862 2,4181 0,39742 0,56329 3385,22 278,06 115,31

    Расчет оконечного полузвена.

    Четвертое звено состоит из двух полузвеньев, включенных на входе и на выходе фильтра. Для того, чтобы рассчитать номинальные величины Г-образных полузвеньев по приведенным формулам, необходимо пересчитать величину , так как оконечные полузвенья образуются путем деления пополам Т-образного звена типа 2А1н (линия а-а на рисунке 11).

    Рисунок 11 – Преобразование звена типа 2А1н в оконечные полузвенья ФНЧ и расчетные соотношения.

    Проверяем резонансную частоту контура

    ()

    Имея полную схему фильтра (см. рисунок 8), можно составить его приведенную схему, отличающуюся от полной тем, что параллельно соединенные элементы заменим одним эквивалентным элементом (см.рисунок 12). Это внесет упрощение в изготовление фильтра и также удешевит его. В приведенной схеме изменены обозначения. Поэтому приведем нумерацию и расчет эквивалентных элементов (обозначения полной схемы в скобках):

    , ,

    ,,

    ,,

    ,,

    ,

    Рисунок 12 – Приведенная схема фильтра нижних частот

    Построение полной характеристики затухания ФНЧ. Необходимо рассчитать затухание в полосе задерживания и в полосе пропуская. Так же учтем, что общее собственное затухание как в полосе задерживания, так и в полосе пропускания равняется сумме затуханий всех звеньев . Поэтому рассчитаем затухания отдельно для каждого звена, и затем просуммируем их.

    Расчет характеристики затухания фильтра нижних частот начинаем с расчета собственного затухания каждого звена на частотах , где учитываем что Q – добротность катушек индуктивности (для выбранных мною альсиферовых сердечников она обычно берется равной 60) и – рассчитанные ранее коэффициенты звеньев.

    Таким образом

    Далее считаем собственное затухание в полосе задерживания без учета потерь, так как потери в полосе задерживания не сказываются существенно на затухании.

    ,

    где

    Рассчитываем собственное затухание в полосе задерживания на всех частотах и , кроме частоты бесконечного затухания данного звена, так как оно рассчитано ранее. Полный расчет для звена I () приведен в таблице 4. Для всех остальных звеньев в таблице 5 приведены расчетные значения частот и и величины соответствующих им затуханий, там же приводится собственное затухание всего ФНЧ в полосе задерживания.

    Таблица 4

    I звено
    f,кГц
    115,00 1,0176 5,4001 6,0296 4,0296 0,40301
    115,313 1,0203 5,0385 5,6928 3,0928 0,43282
    116,419 1,0301 4,1056 4,8239 2,8239 0,5346
    119,137 1,0542 3,1594 3,9426 1,9426 0,7048
    125,179 1,1076 2,3259 3,1663 1,1663 0,9987
    139,356 1,2331 1,7091 2,5918 0,5918 1,4769
    176,369 1,5606 1,3026 2,2132 0,2132 2,3399

    Таблица 5

    f,кГц I звено II звено III звено IV звено
    310,442 9,381 1,47681 0,7077 0,4333 11,99881
    176,369 2,3399 2,00241 0,8765 0,5294 5,74821
    139,356 1,4769 5,247 1,21056 0,7071 8,64156
    125,179 0,9987 1,87846 1,8832 0,9997 5,76006
    119,137 0,7078 1,21113 2,328 1,4738 5,72073
    116,419 0,53546 0,88646 2,0395 2,2865 5,74792
    115,313 0,43282 0,70645 1,4737 3,39 6,00297
    115 0,40301 0,6556 1,34079 1,04941 3,44881

    Для получения рабочего затухания в полосе задерживания рассчитаем затухание задерживания , так как оно существенно влияет в этой области частот на рабочее затухание.

    ,

    где

    Затухание отражения считаем только для оконечного полузвена со значением , так как затухание отражения получается за счет несогласованности характеристического сопротивления с сопротивлением нагрузки. Окончательные результаты сведены в таблице 6.

    Таблица 6

    f, кГц , неп f, кГц , неп
    115,000 –0,5637 125,179 –0,3235
    115,3013 –0,5972 139,356 7,45
    116,419 –0,674 176,369 –0,467
    119,137 –0,6806 310,442 –0,602

    Далее находим рабочее затухание всего фильтра в полосе задерживания (таблица 7).

    Таблица 7

    f, кГц

    ,

    дБ

    ,

    неп

    115,00 1,0176 3,44881 –0,5637 2,88511 28,14 2,89
    115,313 1,0203 6,00297 –0,5972 5,40577 47,07 5,41
    116,419 1,0301 5,74792 –0,674 5,07392 44,11 5,07
    119,137 1,0542 5,72073 –0,6806 5,04013 43,85 5,04
    125,179 1,1076 5,76006 –0,3235 5,43656 47,33 5,44
    139,356 1,2331 8,64156 7,45 16,0916 139,98 16,09
    176,369 1,5606 5,74821 –0,467 5,28121 45,94 5,28
    310,442 2,75 11,99881 –0,602 11,3968 99,15 11,39

    Рассчитываем фазовый сдвиг, вносимый одним звеном ФНЧ.

    где – фазовый сдвиг, вносимый каждым звеном;

    = 1,2,3,4 – № звена

    Общий фазовый сдвиг представляет собой сумму фазовых сдвигов звеньев. Расчеты фазового сдвига каждого звена и всего фильтра приведены в таблицах 8 и 9. Как видно из таблицы 9 фазовый сдвиг вырастает при приближении к частоте среза. Для ФНЧ эта частотой является наивысшей частотой полосы пропускания.

    Для получения полной характеристики затухания ФНЧ рассчитываем для каждого звена в отдельности затухание в полосе пропускания с учетом потерь по следующей формуле:

    Таблица 8

    I звено
    f, кГц f, кГц
    50 0,4424 0,4595 80 0,7079 0,9334
    55 0,4867 0,5189 85 0,7521 1,0629
    60 0,5309 0,5835 90 0,7964 1,2264
    65 0,5751 0,6548 95 0,8406 1,4453
    70 0,6194 0,7348 100 0,8848 1,7688
    75 0,6636 0,8262

    Расчет собственного затухания в полосе пропускания, вносимого I звеном, отражен в таблице 10. Окончательные результаты аналогичных расчетов затухания полосы пропускания для остальных звеньев сведены в таблице 11. В этой же таблице приведено суммарное собственное затухание фильтра в полосе пропускания .

    Таблица 9

    Таблица10

    I звено Q=60
    f, кГц , неп , дБ
    50 0,4424 0,21114 1,21114 0,37939 0,002797 0,02433
    55 0,4867 0,26926 1,6926 0,30657 0,002487 0,02164
    60 0,5309 0,34047 1,3047 0,44723 0,003957 0,03443
    65 0,5751 0,42876 1,42876 0,45829 0,004393 0,03822
    70 0,6194 0,53993 1,53993 0,47716 0,004926 0,04286
    75 0,6636 0,68261 1,68261 0,49102 0,005431 0,04725
    80 0,7079 0,87124 1,87124 0,49881 0,005885 0,05120
    85 0,7521 1,12976 2,12976 0,49907 0,006256 0,05443
    90 0,7964 1,50406 2,50406 0,48976 0,006501 0,05657
    95 0,8406 2,08889 3,08889 0,46790 0,006555 0,05703
    100 0,8848 3,12865 4,12865 0,42842 0,063178 0,05496

    За счет потерь энергии в катушках индуктивности и конденсаторах собственное затухание фильтра не равно нулю, а имеет некоторое конечное значение, возрастающее по мере приближения к предельной частоте.

    На основании ранее полученных данных о затухании фильтра составляем таблицу для расчета выходного напряжения в полосе пропускания и полосе задерживания фильтра нижних частот, помня, что и (таблица 12).

    Рисунок 13 – Фазовая характеристика ФНЧ с полосой

    пропускания 50–100 кГц

    Таблица 12

    f, кГц , дБ , В f, кГц , дБ , В
    50 0,057409 4,967 100 0,217759 4,876
    55 0,062213 4,964 115,00 28,14 0,196
    60 0,083417 4,952 115,313 47,07 0,0222
    65 0,096649 4,945 116,419 44,11 0,03115
    70 0,111872 4,936 119,137 43,85 0,0321
    75 0,128068 4,927 125,179 47,33 0,0215
    80 0,145215 4,917 139,356 139,98 0
    85 0,163141 4,907 176,369 45,94 0,0252
    90 0,181667 4,896 310,442 99,15 0,000055
    95 0,200216 4,886

    Таблица 11

    f, кГц

    I звено

    II звено

    III звено

    IV звено

    ,

    неп

    ,

    дБ

    , неп , дБ , неп , дБ , неп , дБ , неп , дБ
    50 0,002797 0,02433 0,001964 0,01709 0,001124 0,009775 0,000714 0,006214 0,006599 0,057409
    55 0,002487 0,02164 0,00239 0,020792 0,001387 0,012067 0,000887 0,007714 0,007151 0,062213
    60 0,003957 0,03443 0,002859 0,024873 0,001687 0,014681 0,001084 0,009433 0,009587 0,083417
    65 0,004393 0,03822 0,003372 0,029336 0,002031 0,017672 0,001313 0,011421 0,011109 0,096649
    70 0,004926 0,04286 0,003928 0,034175 0,002426 0,021102 0,001579 0,013735 0,012859 0,111872
    75 0,005431 0,04725 0,004522 0,039338 0,002878 0,025037 0,00189 0,016443 0,014721 0,128068
    80 0,005885 0,05120 0,005148 0,044789 0,003399 0,029574 0,002259 0,019652 0,016691 0,145215
    85 0,006256 0,05443 0,005789 0,050363 0,004004 0,034836 0,002703 0,023512 0,018752 0,163141
    90 0,006501 0,05657 0,006419 0,055829 0,004713 0,041 0,003249 0,028268 0,020882 0,181667
    95 0,006555 0,05703 0,006973 0,060658 0,005543 0,048221 0,003943 0,034307 0,023014 0,200216
    100 0,063178 0,05496 0,007332 0,063794 0,00651 0,05664 0,00487 0,042365 0,08189 0,217759

    Рисунок – 14 Амплитудно-частотная характеристика ФНЧ с полосой пропускания 50–100 кГц

    Окончательной проверкой правильности расчета фильтра является близкое совпадение расчетной характеристики с экспериментальной (рисунок 15).

    1–теоретическая характеристика; 2– расчетная характеристика

    Рисунок – 15 Амплитудно-частотная характеристика ФНЧ с полосой пропускания 50–100 кГц

    В заключении расчета приведем общую схему фильтра нижних частот, сопротивление нагрузки ставим в конце схемы.

    ; ; ; ;

    ; ; ; ; ; ; ; ; ;

    Рисунок –16 Общая схема ФНЧ с рассчитанными значениями емкостей и индуктивностей

    Так как в расчете использованы катушки индуктивности с альсиферовыми тороидальными сердечниками, то рекомендуется расположение всех катушек на общей оси и воспользоваться изолирующими прокладками между ними. Монтаж конденсатора может быть осуществлен подобно монтажу сопротивлений, на общей плате. Кожух фильтра выполняется из алюминия, дюраля либо пермаллоя, или другого материала, играющего роль электрического экрана (рисунок 17). Выводы осуществляются через разъемы, которые могут быть и герметичными. Если требуется герметичность всего фильтра, можно рекомендовать заливку компаундами, например эпоксидной смолой. При этом катушки индуктивности должны пройти предварительную пропитку церезином.

    При монтаже всех деталей требуется тщательная и надежная пайка, потому что в большинстве случаев неисправности фильтре связаны с плохой пайкой контактов.

    Рисунок 17 – Наружный вид фильтра нижних частот 120 80 60

    Заключение

    В результате выполнения курсового проекта был рассчитан фильтр нижних частот 2-го порядка по сопротивлению, с полосой пропускания 50–100кГц. Характеристики затуханий, построенные по рассчитанным частотам, отвечают требованиям к полосам задержания и пропускания. Проведенное моделирование показало, что характеристики рассчитанных фильтров близки к идеальным, что подтверждает точность расчёта и возможность использования этого фильтра на практике. Преобразование общей схемы фильтра, т.е. уменьшение общего количества необходимых емкостей и индуктивностей позволило удешевить и уменьшить вес конструкции фильтра.

    Следует отметить, чторассчитанный мною фильтр может использоваться как все L C -фильтры: в силовых электрических цепях для гашения помех и для сглаживания пульсаций напряжения после выпрямителя; в каскадах радиоэлектронной аппаратуры, включенный на входе средневолнового радиоприёмника обеспечивая настройку на определённую радиостанцию; в звуковой аппаратуре в многополосных эквалайзерах для корректировки АЧХ; для разделения сигналов низких, средних и высоких звуковых частот в многополосных акустических системах; в схемах частотной коррекции магнитофонов и др.

    Список использованной литературы

    1. Богданов Н.Г. Расчёт электрических фильтров. – Пособие по курсовому и дипломному проектированию – М.: ВИПС, 2000 г.,352 с.

    2. Зааль Р., Справочник по расчетам фильтров – М.: Радио и связь, 1983 г., 753 с.

    3. Куцко Т. Ю. Расчет полосовых фильтров – М.: Энергия 1985 г., 193 с.

    4. Босый К. Д. Электрические фильтры – М.: Гостехиздат 1960 г., 350 с.

    1.19. RC-фильтры

    ОСНОВЫ ЭЛЕКТРОНИКИ

    Полное и реактивное сопротивление



    Благодаря тому что импеданс конденсатора, равный Zc = -j/ωС, зависит от частоты, с помощью конденсаторов и резисторов можно строить частотно-зависимые делители напряжения, которые будут пропускать только сигналы нужной частоты, а все остальные подавлять. В этом разделе вы познакомитесь с примерами простейших RС-фильтров, к которым мы будем неоднократно обращаться в дальнейшем. В гл. 5 описаны более сложные фильтры.

    Рис. 1.52. Фильтр высоких частот.

    Фильтры высоких частот. На рис. 1.52 показан делитель напряжения, состоящий из конденсатора и резистора. Согласно закону Ома для комплексных величин,

    I = Uвх/Zполн = Uвх/R — (j/ωC) = Uвх[R + j/ωC)]/R2 + 1/ω2C2.

    (Окончательный результат получек после умножения числителя и знаменателя на комплексное число, сопряженное знаменателю.) Итак, напряжение на резисторе R равно

    Uвых = IZR = IR = Uвх[R + (j/ωС)R]/R2+1/ω2C2.

    Чаще всего нас интересует не фаза, а амплитуда Uвых:

    Uвых = (UвыхUвых*)1/2 = UвхR/[R2 + (1/ω2C2)]1/2.

    Сравните полученный результат с выражением для резистивного делителя:

    Uвых = UвхR1/(R1 + R2).

    Векторное представление импеданса RС — цепи (рис. 1.53) показано на рис. 1.54.

    Рис. 1.53.

    Рис. 1.54.

    Итак, если не принимать во внимание сдвиг фаз, а рассматривать только модули комплексных амплитуд, то «отклик» схемы будет определяться следующим образом:

    Uвых = UвхR/[R2 + (1/ω2C2)]1/2 = Uвх2πƒRC/[1 + (2πƒRC)]1/2.

    График этой зависимости представлен на рис. 1.55. Такой же результат мы бы получили, если бы определили отношение модулей импедансов как в упражнении 1.17 и в примере перед этим упражнением; числитель представляет собой модуль импеданса нижнего плеча делителя R, а знаменатель — модуль импеданса последовательного соединения R и С.

    Рис. 1.55. Частотная характеристика фильтра высоких частот.

    Как вы видите, на высоких частотах выходное напряжение приблизительно равно входному (ω > 1/RC), а на низких частотах выходное напряжение уменьшается до нуля. Мы пришли к важному результату, запомните его. Подобная схема, по понятным причинам, называется фильтром высоких частот. На практике ее используют очень широко. Например, в осциллографе предусмотрена возможность связи по переменному току между исследуемой схемой и входом осциллографа. Эта связь обеспечивается с помощью фильтра высоких частот, имеющего перегиб характеристики в области 10 Гц (связь по переменному току используют для того, чтобы рассмотреть небольшой сигнал на фоне большого напряжения постоянного тока). Инженеры часто пользуются понятием «точки излома» -3 дБ для фильтра (или любой другой схемы, которая ведет себя как фильтр)! В случае простого RC — фильтра высоких частот точка излома -3 дБ определяется выражением:

    ƒ3дб = 1/2πRC.

    Обратите внимание, что конденсатор не пропускает ток (ƒ = 0). Самый распространенный пример использования конденсатора-это использование его в качестве блокирующего конденсатора постоянного тока. Если возникает необходимость обеспечить связь между усилителями, то почти всегда прибегают к помощи конденсатора. Например, у любого усилителя звуковой частоты высокого класса все входы имеют емкостную связь, так как заранее не известно, какой уровень постоянного тока будут иметь входные сигналы. Для обеспечения связи необходимо подобрать R и С таким образом, чтобы все нужные частоты (в данном случае 20 Гц — 20 кГц) поступали на вход без потерь (без деления на входе).

    Рис. 1.56. а — Изменение реактивного сопротивления индуктивностей и конденсаторов в зависимости от частоты. Все декады одинаковы и отличаются лишь масштабом. б — Увеличенное изображение одной декады из графика А. график построен для стандартных компонентов, имеющих точность 20%.

    В качестве примера рассмотрим фильтр, показанный на рис. 1.57. Это фильтр высоких частот с точкой перегиба 3 дБ на частоте 15,9 кГц. Импеданс нагрузки, подключаемой к фильтру, должен быть значительно больше 1 кОм. иначе нагрузка будет искажать выходное напряжение фильтра. Источник сигнала должен обеспечивать возможность подключения нагрузки 1 кОм без значительной аттенюапии (потери амплитуды сигнала), иначе фильтр будет искажать выход источника сигнала.

    Рис. 1.57. Рис. 1.58. Фильтр низких частот.

    Фильтры низких частот. Если поменять местами R и С (рис. 1.58), то фильтр будет вести себя противоположным образом в отношении частоты. Можно показать, что Uвых = [1/1 + ω2R2С2)1/2] Uвх. График этой зависимости представлен на рис. 1.59. Такой фильтр называют фильтром низких частот. Точка -3 дБ на характеристике фильтра находится на частоте ƒ = 1/2πRC. Фильтры низких частот находят очень широкое применение. Например, их используют для устранения влияния близлежащих радио — и телевизионных станций (550 кГц — 800 МГц), на работу усилителей звуковых частот и других чувствительных электронных приборов.


    Рис. 1.59 Частотная характеристика фильтра низких частот.

    Упражнение 1.21. Докажите справедливость выражения для выходного напряжения фильтра низких частот.

    Выход фильтра низких частот можно рассматривать в качестве самостоятельного источника сигналов. При использовании идеального источника напряжения переменного тока (с нулевым импедансом) фильтр со стороны выхода низких частот имеет сопротивление R (при расчетах полных сопротивлений идеальный источник сигналов можно заменить коротким замыканием, т.е. его нулевым импедансом для малого сигнала). В выходном импедансе фильтра преобладает емкостная составляюшая. и на высоких частотах он становится равным нулю. Для входного сигнала фильтр представляет собой нагрузку, состоящую на низких частотах из сопротивления R и сопротивления нагрузки, а на высоких частотах — нагрузку, равную просто сопротивлению R.

    Рис. 1.60. Фазочастотная и амплитудно-частотная характеристики фильтра низких частот, изображенные в логарифмическом масштабе. В точке 3 дБ фазовый сдвиг составляет 45° и в пределах декады изменения частоты лежит в пределах 6° от асимптотическою значения.

    На рис. 1.60 изображена также частотная характеристика фильтра низких частот, но в более общепринятом виде для вертикальной и горизонтальной осей использован логарифмический масштаб. Можно считать, что по вертикальной оси откладываются децибелы, а по горизонтальной — октавы (или декады). На таком графике равные расстояния соответствуют равным отношениям величин. В виде графика изображен также фазовый сдвиг, при этом для вертикальной оси (градусы) использован линейный масштаб, а для оси частот-логарифмический. Такой график удобен для анализа частотной характеристики даже в случае значительной аттенюации (справа): целый ряд таких графиков представлен в гл. 5, посвященной изучению активных фильтров. Отметим, что при значительной аттенюации изображенная на графике кривая вырождается в прямую линию с наклоном -20 дБ/декада (инженеры предпочитают выражение « -6 дБ/октава»). Отметим также, что фазовый сдвиг плавно изменяется от 0° (на частотах ниже точки перегиба) до 90° (на частотах существенно выше точки перегиба), а в точке -3 дБ составляет 45°. Практическое правило для односекционных RС — фильтров говорит о том. что фазовый сдвиг составляет ≈6° от асимптот в точках 0.1ƒ3дБ и 10ƒ3дБ.

    Упражнение 1.22. Докажите последнее утверждение.

    Возникает интересный вопрос: можно ли сделать фильтр с какой-либо другой заданной амплитудной характеристикой и какой-либо другой заданной фазовой характеристикой. Пусть вас это не удивляет, но ответить можно только отрицательно — нельзя. Фазовая и амплитудная характеристики для всех возможных фильтров подчиняются законам причинной связи (т.е. характеристика является следствием определенных свойств, но не их причиной).

    Частотные характеристики дифференцирующих и интегрирующих RС — цепей. Схема дифференцирующей RС — цепи, которую мы рассмотрели в разд. 1.14, имеет такой же вид, как и схема фильтра высоких частот, приведенная в настоящем разделе. Чем же считать такую схему, зависит от того, что вас больше интересует: преобразование сигналов во времени или частотная характеристика. Полученное ранее временное условие правильной работы схемы (Uвых « Uвх) можно сформулировать иначе, применительно к частотной характеристике: для того чтобы выходной сигнал был небольшим по сравнению с входным, частота должна быть значительно ниже, чем в точке -3 дБ. В этом легко убедиться. Допустим, что входной сигнал равен Uвх = sinωt. Воспользуемся уравнением, которое мы получили ранее для выхода дифференциатора:

    Uвх = RC d/dt sinωt = ωRCcosωt.

    Отсюда Uвых « Uвх, если ωRC « 1, т.е. RC « 1/ω. Если входной сигнал содержит некоторый диапазон частот, то условие должно выполняться для самых высоких частот входного диапазона.

    Схема интегрирующей RC — цепи (разд. 1.15) имеет такой же вид, как и схема фильтра низких частот: аналогично в хорошем интеграторе самые низкие частоты входного сигнала должны существенно превышать частоту в точке -ЗдБ.

    Индуктивности и конденсаторы. Индуктивности, также как и конденсаторы, в сочетании с резисторами образуют схемы фильтров низких (или высоких) частот. Однако на практике RL — фильтры низких и высоких частот встречаются редко. Это связано с тем, что индуктивности более громоздки и дороги, а работают хуже, чем конденсаторы (их характеристики более существенно отличаются от идеальных). Если есть возможность выбора, то предпочтение лучше отдать конденсатору. Исключением из этой общей рекомендации являются ферритовые бусины (маленькие торроидальные сердечники) и дроссели в высокочастотных схемах. Несколько бусин нанизывают на провод, благодаря этому соединение, выполненное с помощью провода, становится в некоторой степени индуктивным; импеданс на высоких частотах увеличивается и предотвращает «колебания» в схеме, при этом в отличие от RС — фильтра активное сопротивление схемы не увеличивается. Радиочастотный дроссель — это катушка, состоящая из нескольких витков провода и ферритового сердечника и используемая с той же целью в радиочастотных схемах.


    Диоды и диодные схемы


    Полосовой фильтр и режекторный фильтр

    Одноэлементные фильтры высоких и низких частот

    Как правило, одноэлементные фильтры высоких и низких частот применяют непосредственно в акустических системах мощных усилителей звуковой частоты, для улучшения звучания самих звуковых «колонок».

    Они подключаются последовательно с динамическими головками. Во первых, они берегут как динамические головки от мощного электрического сигнала, так и усилитель от низкого сопротивления нагрузки не нагружая его лишними динамиками, на той частоте, которую эти динамики не воспроизводят. Во вторых, они делают воспроизведение приятнее на слух.

    Чтобы рассчитать одноэлементный фильтр, необходимо знать реактивное сопротивление катушки динамической головки. Расчёт производится по формулам делителя напряжения, что так же справедливо для Г-образного фильтра. Чаще всего, одноэлементные фильтры подбирают «на слух». Для выделения высоких частот на «пищалке» последовательно с ней устанавливается конденсатор, а для выделения низких частот на низкочастотном динамике (или сабвуфере), последовательно с ним подключается дроссель (катушка индуктивности). Например, при мощностях порядка 20…50 Ватт, на пищалки оптимально использовать конденсатор на 5…20 мкФ, а в качестве дросселя низкочастотного динамика использовать катушку, намотанную медным эмалированным проводом, диаметром 0,3…1,0 мм на бобину от видеокассеты VHS, и содержащую 200…1000 витков. Указаны широкие пределы, потому, как подбор – дело индивидуальное.

    Фильтры для динамиков своими руками

    Сделать фильтр для динамика совсем не сложно. Он состоит всего из двух элементов – конденсатора и катушки индуктивности. Рассчитать параметры радиоэлементов для пассивной схемы низкой частоты второго порядка проще всего на онлайн калькуляторе. Там можно задать желаемый уровень среза и сопротивление акустической головки. Программа выдаст требуемую ёмкость конденсатора и индуктивность катушки. Например, выбран уровень среза 150 Гц, а сопротивление динамика равно 4 Ом. Калькулятор выдаст следующие значения:

    • Ёмкость конденсатора – 187 мкф
    • Индуктивность катушки – 6,003 мГн

    Требуемую ёмкость можно получить из параллельно соединённых конденсаторов К78-34, которые специально разработаны для работы в акустических системах. Кроме того есть обновлённая линейка конденсаторов аналогичного типа. Это KZKWhiteLine. В качестве недорогих аналогов, радиолюбители часто используют конденсаторы типа МБГО или МБГП.

    Катушка индуктивности на 6 мГн наматывается на оправке диаметром 1 см и длиной 6 см. Поскольку катушка не имеет магнитного сердечника в качестве бобины можно использовать цилиндр из любого материала, на который для удобства намотки, нужно сделать щёчки. Для намотки используется медный провод типа ПЭЛ диаметром 1 мм. Длина проволоки 84 метра. Намотку нужно делать виток к витку.

    Фото фильтров низких частот

    Также рекомендуем просмотреть:

    • Полировка фар своими руками
    • Строительные леса своими руками
    • Точилка для ножей своими руками
    • Антенный усилитель
    • Восстановление аккумулятора
    • Мини паяльник
    • Как сделать электрогитару
    • Оплетка на руль
    • Фонарик своими руками
    • Как заточить нож для мясорубки
    • Электрогенератор своими руками
    • Солнечная батарея своими руками
    • Течет смеситель
    • Как выкрутить сломанный болт
    • Зарядное устройство своими руками
    • Схема металлоискателя
    • Станок для сверления
    • Нарезка пластиковых бутылок
    • Аквариум в стене
    • Врезка в трубу
    • Стеллаж в гараж своими руками
    • Симисторный регулятор мощности
    • Вечный фонарик
    • Нож из напильника
    • Усилитель звука своими руками
    • Трос в оплетке
    • Пескоструйный аппарат своими руками
    • Генератор дыма
    • Ветрогенератор своими руками
    • Акустический выключатель
    • Воскотопка своими руками
    • Туристический топор
    • Стельки с подогревом
    • Паяльная паста
    • Полка для инструмента
    • Пресс из домкрата
    • Золото из радиодеталей
    • Штанга своими руками
    • Как установить розетку
    • Ночник своими руками
    • Аудио передатчик
    • Датчик влажности почвы
    • Счетчик Гейгера
    • Древесный уголь
    • Wi-Fi антенна
    • Электровелосипед своими руками
    • Ремонт смесителя
    • Индукционное отопление
    • Стол из эпоксидной смолы
    • Трещина на лобовом стекле
    • Эпоксидная смола
    • Как поменять кран под давлением
    • Кристаллы в домашних условиях

    Помогите проекту, поделитесь в соцсетях 😉

    Самодельные сетевые фильтры

    Нередко имеющиеся в продаже дешевые фильтры на самом деле фильтрами не являются. Например, фильтр-удлинитель (рис.9). Там внутри находится лишь варистор, ограничивающий кратковременные высоковольтные импульсы, которые иногда возникают в сети, и токовый размыкатель, срабатывающий при протекании большого тока (рис 10).

    Рис. 9. Фильтр-удлинитель.

    Рис. 10. Что внутри фильтра-удлиннителя.

    На корпусе есть кнопка, которую нужно нажать, чтобы снова замкнуть размыкатель, если он сработал. Для превращения этого удлинителя в полноценный фильтр внутрь нужно встроить фильтрующие цепи.

    На исходной схеме (рис.11а) S1 -токовый размыкатель, VR1 — варистор типа 471 (числом кодируется максимальное напряжение, а от диаметра зависит максимальная энергия подавляемого импульса).

    Рис. 11. Схема фильтрующих цепей для встраивания в удлиннитель-розетку.

    В доработанном варианте (рис. 11 б) добавляется RLC-фильтр. Катушки L1 и 12 вместе с конденсаторами С1 и С2 образуют LC-фильтр.

    Индуктивное сопротивление катушек растет на высоких частотах. Чтобы ослабить и низкочастотные помехи, последовательно с катушками включены резисторы R1 и R2. Резистор R3 разряжает конденсаторы при отключении фильтра от сети. При сборке фильтра (рис. 12) варистор оставляется штатный (типа 471, диаметром 6…10 мм).

    Чем больше сопротивление резисторов R1 и R2, тем лучше фильтрация, но больше их нагрев и потери напряжения в фильтре. Поэтому сопротивление резисторов выбирается в зависимости от суммарной мощности, потребляемой всеми теми устройствами, которые будут подключаться к фильтру (при указанных номиналах РНагр.макс=250 Вт).

    Дроссели L1 и L2 — промышленные высокочастотные, типа ДМ-1 индуктивностью 50…100 мкГн. Конденсаторы — пленочные, типа К73-17 или аналогичные (импортные меньше по габаритам) емкостью не менее 0,22 мкФ (больше 1 мкФ тоже не нужно). Сопротивление резистора РЗ — не критично (от 510 кОм до 1,5 МОм).

    Дополнительно на сетевой провод возле самого удлинителя желательно одеть ферритовую шайбу (удобнее всего разрезную на защелках — рис.13).

    Рис. 12Сборка фильтра.

    Рис. 13. Ферритовая шайба.

    Другой вариант схемы помехоподавляющего сетевого фильтра приведен на рис. 14. Для большей эффективности он состоит из двух соединенных последовательно звеньев.

    Первое (конденсаторы С1, С4, С5, С8, С9 и двухобмоточный дроссель 12) отвечает за подавление помех частотой выше 200 кГц.

    Второе звено (двухобмоточный дроссель И с остальными конденсаторами) подавляет помехи, спектр которых простирается ниже указанной частоты (вплоть до единиц килогерц).

    Рис. 14. Схема помехоподавляющего сетевого фильтра.

    Благодаря магнитной связи между обмотками дросселей происходит подавление синфазных помех (тех, что наводятся одновременно на оба сетевых провода или излучаются ими).

    Поэтому обмотки каждого дросселя должны быть одинаковыми и симметрично намотанными на магнитопроводы

    Важно обеспечить правильную фазировку обмоток

    Их начала обозначены на схеме точками. Дроссель L1 намотан на ферритовом магнитопроводе Ш12×14 с самодельным каркасом из злектрокартона сложенным вдвое проводом ПЭЛШО 00,63 мм. Обмотка содержит 87 витков. Марка феррита, к сожалению, неизвестна. Измеренная прибором 1.Р235 индуктивность каждой обмотки — около 20 мГн.

    Для дросселя 1.2 использован броневой магнито-провод Б22 из феррита 2000НМ1. Его обмотки содержат по 25 витков и намотаны тем же проводом и таким же образом, что и обмотки дросселя L1. Индуктивность каждой обмотки дросселя L2 — 120 мкГн.

    Конденсаторы первого звена фильтра — слюдяные. Поскольку малогабаритных конденсаторов такого типа требующейся для фильтра емкости на нужное напряжение не существует, пришлось соединить попарно-параллельно конденсаторы КСО-5 меньшей емкости.

    Аналогичное решение, но с попарно-последовательным соединением конденсаторов С2, С3 и С6, С7 (пленочных зарубежного производства), принято и во втором звене фильтра для обеспечения нужного рабочего напряжения.

    Подключенные параллельно конденсаторам резисторы R1…R4 выравнивают приложенные к ним напряжения и обеспечивают быструю разрядку всех конденсаторов после отключения фильтра от сети. Конденсатор С9 — типа К78-2. Плата фильтра помещена в заземленную металлическую коробку.

    Материал подготовил В. Новиков. РМ-07-12, 08-12.

    Источники информации:

    1. electroclub.info
    2. corumtrage.ru
    3. potrebitel.ru

    Как фильтры LC улучшают работу FPV

    Регуляторы оборотов и моторы генерируют много шумов, которые вы можете наблюдать на экране очков или шлема. Вы, наверное, замечали, что если даете большой газ, становится больше помех, так как двигатели начинают потреблять больше тока, а бОльший ток — это больше создаваемых помех (шумов).

    Шум представляет из себя рябь и белые полосы на видео, вы это точно видели.

    LC-фильтры подавляют такие шумы, за счет внедрения их в цепь питания квадрокоптера, например, на питание камеры или видеопередатчика.

    Некоторые регуляторы оборотов уже продаются с встроенными фильтрами, но они бывают недостаточными, поэтому пилоты ставят еще и LC.

    Сведение фильтров

    Теперь начинается финальный этап — сведение фильтров. Пора намотать катушки… или не намотать? Мотать всегда лень, нет провода, каркасов, конкретных значений индуктивности. В виду этих причин поискав в хламе нашлись пары катушек на 0,8 мкг и 3 мкг — на них и пришлось строить. В крайнем случаи всегда же можно домотать или отмотать лишнее.

    По графику видно, что раздел попал в район 1,8 кгц, что вполне вписывается в задуманные границы. Подбором конденсаторов удалось добиться следующего импеданса. На частоте раздела имеется два бугорка, но их высота меньше полу ома — это не критично. Это не конечный его вид, в последствии был несколько увеличен резистор в цепочке Цобеля пищалки.

    На приведенных выше картинках АЧХ как самого фильтра, так и АЧХ динамиков с его включением.

    Импедансная характеристика динамиков

    Когда с примерными параметрами все более или менее ясно, самое время переходить к практике. Снимаем импедансную характеристику динамиков. С целью оценки сопротивления на графике имеется лесенка с шагом в один Ом. Скачек на 110 герцах это переключение с 10 Ом на 20.

    Разумеется с такими горбами ни один фильтр нормально, и уж тем более расчетно работать не будет, особенно фильтр НЧ. Фильтру ВЧ этот подъем работать в общем то не мешает, однако как упоминалось ранее такой подъем на конце диапазона приведет к подъему высоких частот, в случае если усилитель имеет высокое сопротивление. Это можно использовать и во благо, оставив подъем небольшим.

    Для примерного представления что от чего зависит привожу набор графиков для различных емкостей и сопротивлений. Ступенька начинается с 10 Ом.

    Зная минимальное сопротивление НЧ звена, нужно привести к такому же и ВЧ звено. Тут много вариантов как соединить два резистора и цепочку Цобеля, и каждый кто решился на такой отважный шаг как сведение сам способен определить вид подключения и номиналы резисторов, поэтому описывать данную процедуру здесь излишне. Конкретно в данных колонках по результатам предварительного прослушивания решено было оставить родные резисторы на 2,2 ома и цепочку Цобеля параллельно ВЧ динамику.

    Расчет полосового фильтра

    Расчет полосового фильтра может стать очень сложным занятием даже при использовании операционных усилителей. Тем не менее можно немного упростить методику расчета, и в то же время сохранить производительность полосового фильтра на ОУ на приемлемом уровне.


    Цифровой мультиметр AN8009Большой ЖК-дисплей с подсветкой, 9999 отсчетов, измерение TrueRMS…


    Мультиметр — RICHMETERS RM101Richmeters RM101 — удобный цифровой мультиметр с автоматическим изменен…


    Мультиметр — MASTECH MY68Измерение: напряжения, тока, сопротивления, емкости, частоты…

    Данная схема и методика расчета представляют собой хороший баланс между производительностью и простотой конструкцией фильтра.

    Из рисунка видно, что помимо операционного усилителя схема еще содержит два конденсатора и три резистора.

    Пример упрощенного расчета элементов полосового фильтра на ОУ

    Входные данные:

    • Резонансная частота f = 20Гц.
    • Добротность Q = 10.
    • Коэффициент передачи Hо = 5

    Так как fmax – fmin = f / Q = 2Гц,

    то полоса пропускания составит fmax = 21 Гц, fmin=19 Гц.

    Будем исходить из того, что C1=C2=C=1мкФ

    Тогда сопротивления резисторов можно рассчитать по следующим формулам:

    В нашем случае получим следующие результаты:

    R1 = 10 / (5*2*3,14*20*0,000001) = 15,9 кОм

    R2 = 10 / ((2*10*10-5)*2*3,14*20*0,000001) = 408 Ом

    R3 = 2*10 / (2*3,14*20*0,000001) = 159,2 кОм

    В схеме с одним операционным усилителем, желательно, чтобы коэффициент передачи не превышал 5 и добротность была не более 10. Для получения качественного фильтра параметры резисторов и конденсаторов должны как можно ближе соответствовать расчетным значениям.

    Что такое фильтр?

    Фильтр – это схема, которая удаляет или «отфильтровывает» определенный диапазон частотных компонентов. Другими словами, он разделяет спектр сигнала на частотные составляющие, которые будут передаваться дальше, и частотные составляющие, которые будут блокироваться.

    Если у вас нет большого опыта анализа частотной области, вы можете быть не уверены в том, что представляют собой эти частотные компоненты и как они сосуществуют в сигнале, который не может иметь несколько значений напряжения одновременно. Давайте рассмотрим краткий пример, который поможет прояснить эту концепцию.

    Давайте представим, что у нас есть аудиосигнал, который состоит из идеальной синусоидальной волны 5 кГц. Мы знаем, как выглядит синусоида во временной области, а в частотной области мы не увидим ничего, кроме частотного «всплеска» на 5 кГц. Теперь предположим, что мы включили генератор на 500 кГц, который вносит в аудиосигнал высокочастотный шум.

    Сигнал, видимый на осциллографе, будет по-прежнему представлять собой только одну последовательность напряжений с одним значением на момент времени, но он будет выглядеть по-другому, поскольку его изменения во временной области теперь должны отражать как синусоидальную волну 5 кГц, так и высокочастотные колебания шума.

    Однако в частотной области синусоида и шум являются отдельными частотными компонентами, которые присутствуют одновременно в этом одном сигнале. Синусоидальная волна и шум занимают разные участки представления сигнала в частотной области (как показано на диаграмме ниже), и это означает, что мы можем отфильтровать шум, направив сигнал через схему, которая пропускает низкие частоты и блокирует высокие частоты.

    Рисунок 3 – Представление аудиосигнала и высокочастотного шума в частотной области

    Типы фильтров

    Фильтры, находящие применение в обработке сигналов, бывают

    • аналоговыми и цифровыми
    • пассивными или активными
    • линейными и нелинейными
    • рекурсивными и нерекурсивными

    Среди множества рекурсивных фильтров отдельно выделяют следующие фильтры (по виду передаточной функции):

    • фильтры Чебышёва
    • фильтры Бесселя
    • фильтры Баттерворта
    • эллиптические фильтры

    По порядку (степени уравнения) передаточной функции (см. также ) различают фильтры первого, второго и более высоких порядков. Крутизна ЛАЧХ фильтра 1-го порядка в полосе подавления равна 20 дБ на декаду, фильтра 2-го порядка — 40 дБ на декаду, и т. д.

    По тому, какие частоты фильтром пропускаются (задерживаются), фильтры подразделяются на

    • фильтры нижних частот (ФНЧ)
    • фильтры верхних частот (ФВЧ)
    • полосно-пропускающие фильтры (ППФ)
    • полосно-задерживающие (режекторные) фильтры (ПЗФ)
    • фазовые фильтры

    Сборка

    Давайте рассмотрим инструкцию как правильно сделать простой фильтр

    Для начала следует отшлифовать стеклотекстолит наждачной бумагой и обезжирить. На него способом ЛУТ перенести рисунок платы.

    Возможно потребуется дорисовать лаком дорожки.

    Из лимонной кислоты и перекиси водорода (1:3) готовят раствор для травления. В качестве катализатора используется щепотка соли.

    Заготовленную плату помещают в готовый раствор. После растворения лишней меди промыть плату под проточной водой. Тонер удаляют ацетоном.

    Согласно схеме припаивают детали. Сзади закрепляют перемычку.

    Такая схема работает без настройки. При отсутствии звука проверяют все соединения и крутят резистор.

    Инструкция как правильно сделать простой фильтр довольно проста.

    В нем используют не менее трех конденсаторов. Тетроды помогут уменьшить чувствительность. Их стоимость довольно высока, но они позволяют заметно улучшить качество сигнала.

    Оцените статью:

    Проектирование и расчеты схемы простого LC фильтра верхних частот »Электроника

    Соображения по конструкции, схема и формулы для базового 3-полюсного LC-фильтра верхних частот для ВЧ приложений.


    Постоянный фильтр K Включает:
    Постоянный фильтр k Простая конструкция LC LPF Конструкция LC HPF Конструкция полосового фильтра LC

    Основы фильтра включают: : RF фильтры — основы Характеристики фильтра Основы проектирования ВЧ-фильтров Конструкция фильтра высоких и низких частот Постоянный k-фильтр Фильтр Баттерворта Чебычевский фильтр Фильтр Бесселя Эллиптический фильтр


    Фильтры верхних частот, и в частности LC фильтры верхних частот, используются во многих радиочастотных приложениях, где они блокируют более низкие частоты и пропускают более высокочастотные сигналы.

    Хотя LC фильтры верхних частот не так широко используются, как фильтры нижних частот, они используются во многих областях проектирования ВЧ для удаления нежелательных сигналов и пропуска нежелательных сигналов.

    Обычно LC-фильтры используются для более высоких радиочастот, где активные фильтры не так удобны, а катушки индуктивности более подходят. В низкочастотных конструкциях, как правило, не используются индукторы, так как активные конструкции более распространены, а индукторы могут стать большими и дорогими.

    Топологии фильтра верхних частот

    Фильтры верхних частот с использованием ЖК-компонентов, т.е.е. катушки индуктивности и конденсаторы или даже резисторы и конденсаторы могут быть скомпонованы в простой пи- или Т-схеме. Как следует из его названия, основной элемент π-сети имеет один последовательный конденсатор, а по обе стороны от него есть индуктор, подключенный к земле. Дополнительные элементы сети могут быть включены в каскад, если требуется более высокая скорость спада.

    Топология 3-полюсного π LC-фильтра верхних частот ВЧ

    Аналогично, сетевой фильтр верхних частот T имеет одну индуктивность, соединенную с землей, и с каждой стороны имеется последовательно включенный конденсатор.Как и в случае сети с π-секциями, дополнительные элементы могут быть соединены каскадом для улучшения характеристик спада.

    Топология 3-полюсного ВЧ-фильтра верхних частот T LC

    Таким образом, эти фильтры пропускают высокочастотные сигналы и отклоняют низкочастотные сигналы. Эти фильтры могут использоваться в приложениях, где есть нежелательные сигналы в полосе частот ниже частоты среза, и необходимо пропустить полезные сигналы в полосе выше частоты среза фильтра.

    Конструкция ЖК-фильтра высоких частот

    Существует множество различных вариантов фильтров, которые могут использоваться в зависимости от требований с точки зрения внутриполосной пульсации, скорости, при которой достигается окончательный спад и т. Д.Используемый здесь тип — константа-k, и это дает некоторые управляемые уравнения:

    Сечение π может быть вычислено из приведенных ниже уравнений и с использованием множителей, показанных на диаграмме, то есть 2L и C.

    3-полюсный ЖК-фильтр верхних частот

    Т-образный фильтр верхних частот может быть спроектирован с использованием приведенных ниже уравнений для расчета L и C, но обратите внимание, что диаграмма показывает, что это необходимо масштабировать, поскольку для данной проектной конфигурации требуются 2C и L.

    3-полюсный ВЧ-фильтр верхних частот T LC

    Значения C и L, используемые в двух конструктивных конфигурациях LC-фильтра верхних частот, можно рассчитать с помощью приведенных ниже уравнений.

    L = Z04 π fc Генри

    C = 14 Z0 π fc Фарад

    Fc = 14 π L C Герц

    Где
    Z 0 = характеристическое сопротивление в Ом
    C = Емкость в Фарадах
    L = Индуктивность в Генри
    f c = Частота среза в Герцах

    Конструкция фильтра и меры предосторожности при изготовлении

    Есть несколько рекомендаций, советов и подсказок, которые можно включить в конструкцию и конструкцию фильтра высоких частот, чтобы гарантировать, что он соответствует требованиям конструкции и максимально приближен к теоретическим характеристикам.

    • Каскадирование секций фильтра для лучшего затухания: Чтобы обеспечить больший наклон или спад в фильтре высоких частот, можно каскадировать несколько секций фильтра. При этом фильтрующие элементы из соседних секций можно комбинировать. Например, если два Т-образных фильтра соединены каскадом, и каждая Т-образная секция имеет индуктор 1 мкГн в каждой ветви Т, их можно объединить в смежных секциях и использовать индуктор 2 мкГн.
    • Используйте компоненты с жесткими допусками: Выбор компонентов для любого фильтра, и в данном случае для фильтра верхних частот, важен.Следует использовать компоненты с жесткими допусками, чтобы гарантировать получение требуемых характеристик. Также необходимо проверить температурную стабильность, чтобы убедиться, что компоненты фильтра не изменяются значительно в зависимости от температуры, что приводит к изменению рабочих характеристик.
    • Расположение фильтра: Следует соблюдать осторожность при расположении фильтра, особенно когда фильтр используется для высоких частот. Емкостная и индуктивная связь являются основными элементами, которые ухудшают характеристики фильтра.Соответственно, вход и выход фильтра должны быть разделены. Следует использовать короткие провода и дорожки. Компоненты из соседних секций фильтра должны быть разнесены. При необходимости используются экраны, а на входе и выходе используются качественные разъемы и коаксиальный кабель, если применимо.

    Эта конструкция обеспечивает простой и управляемый способ реализации ЖК-фильтра верхних частот. С помощью простых уравнений можно рассчитать и реализовать проект.

    Другие важные темы по радио:
    Радиосигналы Типы и методы модуляции Амплитудная модуляция Модуляция частоты OFDM ВЧ микширование Петли фазовой автоподстройки частоты Синтезаторы частот Пассивная интермодуляция ВЧ аттенюаторы RF фильтры Радиочастотный циркулятор Типы радиоприемников Радио Superhet Избирательность приемника Чувствительность приемника Обработка сильного сигнала приемника Динамический диапазон приемника
    Вернуться в меню тем радио.. .

    RL high pass, онлайн калькулятор


    Калькулятор и формулы для расчета параметров ВЧ КЛ

    RL онлайн-калькулятор высоких частот

    Эта функция вычисляет свойства фильтра верхних частот, состоящего из резистора и катушки. Выходное напряжение, затухание и фазовый сдвиг рассчитываются для данной частоты.

    \ (\ Displaystyle L \) = Индуктивность [H]

    \ (\ Displaystyle R \) = Сопротивление [Ом]

    \ (\ displaystyle U_1 \) = Входное напряжение [В]

    \ (\ displaystyle U_2 \) = Выходное напряжение [В]

    \ (\ Displaystyle X_L \) = Реактивное сопротивление [Ом]

    \ (\ displaystyle f_g \) = частота среза [Гц]

    \ (\ Displaystyle φ \) = фазовый угол [°]


    Формулы для фильтра высоких частот RL

    Рассчитать коэффициент напряжений

    Выходное напряжение U 2 фильтра верхних частот RL рассчитывается по следующей формуле.2}} \)

    \ (\ Displaystyle X_L = 2 π · е · L \)

    Затухание в децибелах

    На резонансной частоте затухание составляет 3 дБ. Его можно рассчитать для различных частот, используя приведенные ниже формулы. Если входное и выходное напряжение известны, затухание можно легко рассчитать по следующей формуле.

    \ (\ Displaystyle V_u = 20 · lg \ влево (\ гидроразрыва {U_2} {U_1} \ right) \)

    Если напряжения неизвестны, используется следующая формула.2}} \ справа) \)

    Фазовый сдвиг

    В проходе верхних частот RL выходное напряжение опережает входное напряжение на 0 ° — 90 °, в зависимости от частоты. На резонансной частоте фазовый сдвиг составляет 45 °. На частотах выше частоты среза она стремится к 0. На более низких частотах фаза сдвигается в сторону 90 °. Фазовый сдвиг можно рассчитать по следующей формуле.

    \ (\ Displaystyle φ = acos \ влево (\ гидроразрыва {U_2} {U_1} \ вправо) \)
    \ (\ Displaystyle φ = arctan \ влево (\ гидроразрыва {R} {ω · L} \ вправо) \)

    Частота среза

    На частоте среза f g или ω g значение амплитудно-частотной характеристики (т.е. величина передаточной функции) равна 0.707, г. что соответствует -3 дБ.

    \ (\ Displaystyle 0.707 = \ гидроразрыва {1} {\ sqrt {2}} \)

    Формулы частоты среза
    \ (\ Displaystyle ω_g = \ гидроразрыва {R} {L} ⇒ f_g = \ frac {R} {2 · π · L} \)
    \ (\ Displaystyle R = 2 · π · f_g · L \)
    \ (\ Displaystyle L = \ гидроразрыва {R} {2 · π · f_g} \)

    Эта страница полезна? да Нет

    Спасибо за ваш отзыв!

    Прошу прощения за это

    Как мы можем это улучшить?

    послать

    Фильтр высоких частот — моделирование, калькуляторы и теория


    Фильтры высоких частот (HPF) — это схемы, которые позволяют более высоким частотам проходить через них, блокируя более низкие частоты.Это может быть полезно в некоторых приложениях, таких как фильтрация нижних басовых частот музыкального сигнала в высокочастотный динамик (динамик, предназначенный для воспроизведения высоких частот). Кроме того, эти фильтры могут удалять из схемы нежелательные сигналы постоянного тока, обеспечивая связь по переменному току. Пример этого можно найти в опции связи по переменному току осциллографа. По сути, это фильтр верхних частот, настроенный на блокировку очень низких частот, в основном 0 Гц, то есть постоянного тока. Другим примером этого может быть случай, когда вы генерируете синусоидальную волну, используя колебательный контур, который выдает значения только в положительном диапазоне, но вы хотите удалить смещение постоянного тока и отправить сигнал переменного тока, колеблющийся около 0, на динамик.


    RC-фильтр верхних частот

    RC-фильтр верхних частот

    Простейшей схемой HPF является RC-фильтр верхних частот. Этот фильтр высоких частот представляет собой просто конденсатор, соединенный последовательно с резистором, где выходной сигнал находится между двумя компонентами.

    Основная идея того, как работает этот фильтр, заключается в том, что на высоких частотах конденсатор C_1 действует как короткое замыкание, так что весь входной сигнал присутствует на выходе. И наоборот, на низких частотах конденсатор C_1 действует как разомкнутый, эффективно заземляя выход и, следовательно, делая выход равным нулю.

    Ниже приведено видео, в котором мы демонстрируем, как выполнить KiCad-симуляцию RC-цепи HPF.


    Моделирование RC-фильтра верхних частот (видео)

    Это видео моделирования высокочастотного фильтра KiCad, демонстрирующее эффекты регулировки резистора и конденсатора в RC-фильтре верхних частот.


    Калькулятор RC фильтра верхних частот

    резистор ΩkΩMΩ
    конденсатор pFuFF
    Vin (опционально)
    по умолчанию X по умолчанию Y


    Теория (приближение)

    Для аппроксимации RC HPF просто используйте X_ {C_1} для конденсатора и относитесь к нему как к резистору.Ниже приведено уравнение для делителя напряжения

     V_ {out} = V_ {in} {R_1 \ over {R_1 + X_ {C_1}}} 
     \ boxed {V_ {out} = V_ {in} {R_1 \ over {R_1 + {1 \ over (2 \ pi f C_1)}}}} 

    На высоких частотах

    Используя уравнение емкостного реактивного сопротивления, мы можем увидеть, что как \ uparrow f \ Rightarrow {1 \ over (2 \ pi (f \ uparrow) C_1)} \ downarrow \ Rightarrow X_ {C_1} \ downarrow и при достаточно высокой частоте X_ {C_1} \ приблизительно 0, когда это происходит, уравнение RC HPF превращается в V_ {out} \ приблизительно V_ {in} таким образом, все высокие частоты

     V_ {out} = V_ {in} {R_1 \ over R_1 + {1 \ over (2 \ pi {f} C_1) \ uparrow} \ downarrow} \ uparrow 
     V_ {out} \ приблизительно V_ {in} {R_1 \ over R_1 + 0} \ приблизительно V_ {in} \ cancel {R_1 \ over R_1} \ приблизительно V_ {in} 
     \ в штучной упаковке {V_ {out} \ приблизительно V_ {in}} 

    при низкие частоты

    И наоборот, при уменьшении частоты \ downarrow f \ Rightarrow {1 \ over (2 \ pi (f \ downarrow) C_1)} \ uparrow \ Rightarrow X_ {C_1} \ uparrow и при достаточно низкой частоте cy X_ {C_1} \ приблизительно \ infty, когда это происходит, уравнение фильтра верхних частот RC превращается в V_ {out} \ приблизительно 0, таким образом блокируя все низкие частоты, включая DC

     V_ {out} = V_ {in} {R_1 \ over R_1 + {1 \ over (2 \ pi f C_1) \ downarrow} \ uparrow} \ downarrow 
     V_ {out} \ приблизительно V_ {in} {R_1 \ over R_1 + \ infty} \ приблизительно V_ {in} 0 \ приблизительно 0 
     \ в штучной упаковке {V_ {out} \ приблизительно 0} 

    Теория (Комплексное решение)

    В комплексном решении используется X_ {C_1} = {j \ over {w C _1}} вместо X_ {C_1} = { 1 \ over {w C _1}}, где w = 2 \ pi f

     V_ {out} = V_ {in} {R_1 \ over {R_1 - {j \ over {w C_1}}}} = V_ {in} {1 \ over {1 - {j \ over {R_1 w C_1}}}} 

    взяв абсолютное значение комплексного числа, мы получим его величину.{1 \ over {2}}}}}


    Области применения

    Связь по переменному току

    Осциллографы имеют опции для удаления смещения постоянного тока сигнала путем включения связи по переменному току. Это просто фильтр верхних частот, потому что самая низкая частота равна 0 Гц, и это постоянный ток. Таким образом, можно использовать фильтр высоких частот с очень низкой частотой среза для удаления составляющей постоянного тока, обеспечивая, таким образом, связь по переменному току.

    Блокировка постоянного тока

    Другой способ посмотреть на это — конденсаторы блокируют постоянный ток и позволяют переменному току проходить через них.Чем выше частота, тем меньшее реактивное сопротивление (учитывая сопротивление переменному току) обеспечивает конденсатор.

    По мере увеличения частоты реактивное сопротивление конденсатора уменьшается. Это означает, что на низких частотах будет большое реактивное сопротивление, тогда как на высоких частотах реактивное сопротивление будет практически отсутствовать.

     \ downarrow X_ {C_1} = {1 \ over (2 \ pi (f \ uparrow) C_1)} \ downarrow 

    Соответствующие

    Фильтры низких частот

    полоса пропускания — Как возникает формула для расчета низких и высоких частот в полосовом фильтре?

    Общий подход

    Для начала, я уверен, что вы хорошо знакомы с этой конфигурацией инвертирующего усилителя.И я уверен, что вы знаете, что для простых сопротивлений передаточная функция не сложнее, чем:

    $$ G_s = \ frac {V_ \ text {o}} {V_ \ text {i}} = — \ frac {R_ \ text {feedback}} {R_ \ text {source}} $$

    Итак, в вашем случае \ $ R_ \ text {feedback} \ $ и \ $ R_ \ text {source} \ $ вместо этого становятся \ $ Z_ \ text {feedback} \ $ и \ $ Z_ \ text {source} \ $ . Итак:

    $$ G_s = \ frac {V_ \ text {o}} {V_ \ text {i}} = — \ frac {Z_ \ text {feedback}} {Z_ \ text {source}} $$

    На данный момент это просто «заполнить пробелы».2+ \ left (\ frac1 {R_1 \, C_1} + \ frac1 {R_2 \, C_2} \ right) s + \ frac {1} {R_1 \, C_1 \, R_2 \, C_2}} $$

    Установить \ $ \ alpha = \ frac12 \ left (\ frac1 {R_1 \, C_1} + \ frac1 {R_2 \, C_2} \ right) \ $, \ $ \ omega _ {_ 0} = \ frac1 {\ sqrt {R_1 \, C_1 \, R_2 \, C_2}} \ $ и создайте безразмерный \ $ \ zeta = \ frac {\ alpha} {\ omega _ {_ 0}} \ $. 2} \ end {array} \ right.\ end {align *} $$

    \ $ \ zeta \ $ пригодится. Возможны следующие случаи (если вы посмотрите на член квадратного корня из \ $ s_1 \ $ и \ $ s_2 \ $, вы можете заметить, что он может быть воображаемым или реальным):

    $$ \ begin {align *} \ text {Условия коэффициента демпфирования} \ left \ {\ begin {array} {l} \ zeta = 1 \ left (\ alpha = \ omega_0 \ right) && \ text {Критическое демпфирование} \\\ zeta \ gt 1 \ left (\ alpha \ gt \ omega_0 \ right) && \ text {Over-damped} \\\ zeta \ lt 1 \ left (\ alpha \ lt \ omega_0 \ right) && \ text { Недозатухание} \\\ zeta = 0 && \ text {Незатухание} \ end {array} \ right.2 \ $.)

    Обратите внимание, что предыдущая передаточная функция — это всего лишь один из стандартных способов ее записи. Это не единственная стандартная форма. Другой подход — просто заменить \ $ s \ $ на \ $ j \, \ omega \ $ (мы предполагаем, что он не выходит из-под контроля или не превращается в ничто — короче говоря, мы предполагая \ $ \ sigma = 0 \ $.) Кроме того, поскольку \ $ s_1 \ $ и \ $ s_2 \ $ являются действительными корнями (для полосового фильтра с избыточным демпфированием по определению), мы можем переупорядочить следующим образом :

    $$ \ begin {align *} G_s & = — K \: \ left [\ frac {2 \ zeta \, \ omega _ {_ 0} \, s} {\ left (s-s_1 \ right) \ cdot \ left (s-s_2 \ right)} \ right ] \\\\ & = — K \: \ left [\ frac {2 \ zeta \, \ omega _ {_ 0} \, j \, \ omega} {\ left (j \, \ omega + \ omega _ {_ \ text {L}} \ справа) \ cdot \ left (j \, \ omega + \ omega _ {_ \ text {H}} \ right)} \ right] \\\\ & = — K \: \ left [\ frac {2 \ zeta \, \ frac {j \, \ omega} {\ omega _ {_ 0}}} {\ left (1+ \ frac {j \, \ omega} { \ omega _ {_ \ text {L}}} \ right) \ cdot \ left (1+ \ frac {j \, \ omega} {\ omega _ {_ \ text {H}}} \ right)} \ right] \ end {align *} $$

    Дело в том, что есть разные способы изобразить одно и то же.Ваш выбор будет зависеть от того, что вы хотите подчеркнуть. (И, конечно же, стоит потратить время на то, чтобы немного поиграть с уравнениями, чтобы увидеть, куда они вас приведут.)

    (Между прочим, вышеупомянутый относится только к к случаю, когда мы говорим о полосовом фильтре с избыточным демпфированием. Я сделал некоторые предположения о том, что у нас есть реальные и отчетливые корни для этой последней разработки.)

    Проверка с использованием бетонного корпуса

    Давайте просто быстро спроектируем что-нибудь, в основном наугад , и посмотрим, насколько хорошо мы можем предсказать результаты, прежде чем тестировать это, используя вышеупомянутые концепции.

    Я собираюсь просто произвольно выбрать \ $ R_1 = 10 \: \ text {k} \ Omega \ $ и \ $ C_1 = 100 \: \ text {nF} \ $. (Это просто стандартные значения, которые пришли в голову первыми.) Теперь я собираюсь сделать \ $ C_2 = 2.2 \: \ mu \ text {F} \ $, потому что я знаю, что хочу, чтобы он передавал некоторые низкие частоты (я все-таки надеюсь сделать полосу пропускания!) И давайте, наконец, выберем \ $ \ zeta = 2 \ $ … просто потому, что.

    Это оставляет мне необходимость выяснить \ $ R_2 \ $. Если вы используете sympy для такого же мошенничества, как и я, то вы обнаружите, что \ $ R_2 \ приблизительно 6331 \: \ Omega \ $.(Я думаю, вы можете понять, как это решить из предыдущих уравнений, и для этого не нужно держать руку на пульсе.)

    Итак. У меня схема !!!

    смоделировать эту схему — Схема создана с помощью CircuitLab

    Быстрый расчет теперь говорит мне, что \ $ K \ приблизительно 1.474 \ $ или, короче говоря, полоса пропускания будет \ $ \ приблизительно +3.368 \: \ text {dB} \ $. (Обратите внимание, что \ $ K \ $ может быть близким к \ $ \ frac {R_1} {R_2} \ $, но не обязательно точно это значение.)

    Я также быстро вижу, что \ $ \ omega _ {_ 0} = 267.95 \ $ (или \ $ f \ приблизительно 42.65 \: \ text {Hz} \ $), что \ $ \ omega _ {_ \ text {L}} = 71.80 \ $ (или \ $ f _ {_ \ text {L }} \ приблизительно 11,43 \: \ text {Hz} \ $), и что \ $ \ omega _ {_ \ text {H}} = 1000 \ $ (или \ $ f _ {_ \ text {H}} \ приблизительно 159,16 \: \ text {Hz} \ $.)

    (Обратите внимание, что \ $ f _ {_ \ text {L}} \ $ также \ $ \ frac1 {2 \ pi \, R_2 \, C_2} \ $ и что \ $ f _ {_ \ text {H}} \ $ также \ $ \ frac1 {2 \ pi \, R_1 \, C_1} \ $.)

    Вот результаты моделирования LTspice:

    9.3: Фильтры верхних частот — Workforce LibreTexts

    Задача фильтра верхних частот прямо противоположна фильтру нижних частот: обеспечить легкое прохождение высокочастотного сигнала и затруднение перехода к низкочастотному сигналу.Как и следовало ожидать, индуктивная (рисунок ниже) и емкостная (рисунок ниже) версии фильтра верхних частот прямо противоположны их соответствующим конструкциям фильтра нижних частот:

    Емкостной фильтр верхних частот.

    Импеданс конденсатора

    Импеданс конденсатора (рисунок выше) увеличивается с уменьшением частоты. (Рисунок ниже) Этот высокий последовательный импеданс имеет тенденцию блокировать попадание низкочастотных сигналов на нагрузку.

    Отклик емкостного фильтра верхних частот увеличивается с увеличением частоты.

    Индуктивный фильтр верхних частот.

    Импеданс индуктора

    Импеданс катушки индуктивности (рисунок выше) уменьшается с уменьшением частоты. (Рисунок ниже) Этот низкий импеданс при параллельном подключении имеет тенденцию к короткому замыканию низкочастотных сигналов, попадающих на нагрузочный резистор. Как следствие, большая часть напряжения падает на последовательном резисторе R 1 .

    Отклик индуктивного фильтра верхних частот увеличивается с увеличением частоты.

    На этот раз емкостная конструкция является самой простой, требующей только один компонент сверх нагрузки. И, опять же, реактивная чистота конденсаторов по сравнению с катушками индуктивности имеет тенденцию способствовать их использованию в конструкции фильтров, особенно с фильтрами высоких частот, где высокие частоты обычно вызывают странное поведение катушек индуктивности из-за скин-эффекта и электромагнитных потерь в сердечнике.

    Как и фильтры нижних частот, фильтры верхних частот имеют номинальную частоту среза , выше которой выходное напряжение увеличивается выше 70.7% входного напряжения. Так же, как и в случае схемы емкостного фильтра нижних частот, частота среза емкостного фильтра верхних частот может быть найдена по той же формуле:

    В схеме примера нет другого сопротивления, кроме нагрузочного резистора, так что это значение R в формуле.

    Используя стереосистему в качестве практического примера, конденсатор, подключенный последовательно с высокочастотным динамиком, будет служить фильтром верхних частот, придавая высокий импеданс низкочастотным басовым сигналам, тем самым предотвращая расход энергии на динамик неэффективен для воспроизведения таких звуков.Точно так же индуктор, соединенный последовательно с низкочастотным динамиком, будет служить фильтром нижних частот для низких частот, которые конкретный динамик предназначен для воспроизведения. В этой простой примерной схеме среднечастотный динамик подвергается воздействию полного спектра частот с выхода стерео. Иногда используются более сложные сети фильтров, но это должно дать вам общее представление. Также имейте в виду, что я показываю вам только один канал (левый или правый) на этой стереосистеме.Настоящая стереосистема будет иметь шесть динамиков: 2 низкочастотных динамика, 2 среднечастотных динамика и 2 высокочастотных динамика.

    Фильтр верхних частот направляет высокие частоты к высокочастотному динамику, а фильтр нижних частот направляет низкие частоты к низкочастотному динамику.

    Для лучшей производительности нам может потребоваться какая-то схема фильтра, способная передавать частоты между низкими (басы) и высокими (высокие) в среднечастотный динамик, чтобы ни одна из мощностей низкочастотного или высокочастотного сигнала не передавалась. потрачено на громкоговоритель, неспособный эффективно воспроизводить эти звуки.То, что мы будем искать, называется полосовым фильтром , который является темой следующего раздела.

    Обзор

    • Фильтр высоких частот позволяет легко пропускать высокочастотные сигналы от источника к нагрузке и затруднять прохождение низкочастотных сигналов.
    • Емкостные фильтры верхних частот включают конденсатор последовательно с нагрузкой; В индуктивных фильтрах верхних частот резистор включен последовательно, а индуктор — параллельно нагрузке. Первая конструкция фильтра пытается «заблокировать» нежелательный частотный сигнал, а вторая пытается его сократить.
    • Частота среза для фильтра верхних частот — это частота, при которой выходное (нагрузочное) напряжение равно 70,7% входного (исходного) напряжения. Выше частоты среза выходное напряжение превышает 70,7% входного, и наоборот.

    Фильтры — Практический EE

    Мы обсудили ряды Фурье и то, как электромагнитные сигналы можно рассматривать как совокупность частотных компонентов. Фильтр — это схема, которая ослабляет компоненты сигнала в зависимости от их частот.Фильтры характеризуются диапазоном частот, на который они влияют, и их порядком, который представляет собой степень ослабления.

    Диапазоны частот фильтров

    Low-Pass Ослабляет частотные компоненты выше определенного порога
    High-Pass Ослабляет частотные компоненты ниже определенного порога
    Bandpass Проходит непрерывный диапазон (полосу) частот и ослабляет частоты вне этой полосы
    Band-Reject или Notch Ослабляет полосу частот

    Конструкции фильтров

    Пассивный Состоит из пассивных компонентов (резисторы, конденсаторы и катушки индуктивности)
    Активный Содержит активные компоненты, такие как транзисторы и операционные усилители.
    Цифровой Сигналы оцифровываются и обрабатываются как наборы цифровых значений с помощью логических схем или микропрограмм, запущенных на процессоре.

    Пассивные фильтры

    Пассивные фильтры состоят из пассивных компонентов: резисторов, конденсаторов и катушек индуктивности. Быстрый способ определить, какой диапазон частот будет ослабляться расположением пассивных компонентов, — это учесть, что конденсаторы действуют как короткое замыкание для высокочастотных компонентов и действуют как разомкнутые цепи для низкочастотных компонентов.Индукторы все наоборот. И резисторы одинаково влияют на все частотные составляющие. Посмотрите схему ниже.

    Пассивный фильтр нижних частот

    Пассивный фильтр нижних частот

    Схема имеет только один реактивный компонент, конденсатор C1, а R1 и C1 образуют делитель напряжения. По формуле делителя напряжения Vout = Vin * Z C1 / (Z R1 + Z C1 ). Итак, передаточная функция Vout / Vin равна:

    Конденсатор C1 имеет высокое сопротивление по отношению к низкочастотным компонентам.Таким образом, на низкой частоте его импеданс будет большим, что приведет к большому значению передаточной функции. Это означает, что фильтр пропускает низкочастотные составляющие. На высокой частоте конденсатор имеет тенденцию к короткому замыканию, поэтому значение передаточной функции будет минимальным на высокой частоте, что означает, что фильтр ослабляет высокочастотные компоненты. Это фильтр нижних частот.

    А теперь рассмотрим подробнее. Импеданс конденсатора составляет 1 / сС, а сопротивление резистора — R.

    Фильтр нижних частот в частотной области

    Тогда передаточная функция Vout / Vin равна:

    .

    Обратите внимание на пару моментов об этой функции передачи. Во-первых, оно не может быть больше единицы, потому что сопротивление и емкость компонентов являются положительными числами. Когда передаточная функция имеет диапазон значений 1 или меньше, она только ослабляется, а не усиливается. Это всегда будет иметь место с пассивными фильтрами, они только ослабляют, а их передаточные функции всегда имеют значение меньше 1.

    Во-вторых, обратите внимание, что числитель является константой, а знаменатель — многочленом одного порядка от s. Это означает, что это фильтр 1-го порядка, поскольку порядок фильтра — это наивысший порядок полинома (наивысшая степень s) либо в числителе, либо в знаменателе его передаточной функции. Передаточные функции схем, состоящих из пассивных компонентов, всегда будут отношениями полиномов. Этот фильтр не имеет нулей и имеет один полюс -1 / RC, потому что это значение s, которое делает знаменатель равным нулю.

    Порядок фильтра: наивысший полиномиальный порядок числителя или знаменателя передаточной функции

    Как я сказал ранее, полюса и нули передаточной функции схемы являются ключевыми для поведения схемы, и эти значения можно использовать как ярлыки для работы со схемой. Полюс на -1 / RC определяет пороговую частоту, выше которой этот фильтр ослабляет частотные составляющие. Другими словами, он определяет частоту среза, которая является разделительной линией между полосой пропускания и полосой заграждения.Частота среза в радианах — это просто абсолютное значение полюса.

    Частота среза:

    Анализ переменного тока с фильтром нижних частот

    Анализ переменного тока — это анализ частотной характеристики системы, которая является постоянной, а не временной. Математически анализ переменного тока касается части переменной Лапласа (s = σ + jω) , а не σ , которая связана с переходной характеристикой.Подставьте вместо s в передаточной функции, чтобы определить частотную характеристику. Затем преобразуйте в полярную форму, чтобы получить величину и фазу передаточной функции.

    Частотная характеристика: замените
    на s в передаточной функции

    На практике мы используем компьютерный инструмент для выполнения анализа переменного тока. SPICE — это стандартизированная платформа для моделирования схем, и существует пара отличных бесплатных программ SPICE. Я настоятельно рекомендую вам установить оба из них и потратить время на их тщательное изучение.

    программ SPICE:

    Я буду использовать LT Spice для анализа фильтра нижних частот. Анализ переменного тока в SPICE создает график частотной характеристики передаточной функции, показывающий ее величину и фазу. Масштаб может показаться странным. Горизонтальная шкала является логарифмической в ​​декадах (субтики представляют x1, x2,…, x9 и расположены неравномерно). Вертикальная шкала величины также логарифмическая, но в децибелах (дБ). Децибелы для передаточных функций, включающих напряжение и / или ток, равны 20 • log (H (s)), где H (s) — передаточная функция.Децибелы для передаточных функций, включающих мощность, равны 10 • log (H (s)). Вертикальная шкала фазы является линейной и выражается в градусах.

    Горизонтальная шкала анализа переменного тока: логарифмическая шкала в десятилетиях, единицы: Гц.
    Анализ амплитуды переменного тока Вертикальная шкала: 20 • log (H (s)), единицы: дБ.
    Вертикальная шкала фазы анализа переменного тока: линейная шкала, единицы: градусы.
    Анализ переменного тока с фильтром нижних частот

    Пунктирная линия — фаза, а сплошная линия — величина. Этот фильтр нижних частот имеет резистор 10 Ом и конденсатор 100 мкФ, поэтому частота среза составляет 1 / (2 * pi * R * C) = 159 Гц.Частота среза также называется частотой 3 дБ, потому что на этой частоте величина на 3 дБ ниже максимального значения (3 дБ = 1 / sqrt (2) или 0,707). Частота среза находится где-то между 100 Гц на графике и следующей субтикой, и да, действительно, величина там действительно составляет около -3 дБ. И фаза составляет -45 градусов на частоте среза, что означает, что выходной сигнал будет задержан на 45 градусов относительно входного.

    При частоте среза фильтра низких частот:

    — Величина составляет 3 дБ (или.707) ниже макс.
    — Фаза -45 градусов.

    Давайте проведем анализ переходных процессов, чтобы увидеть, что делает вывод в реальном времени. Я настрою функцию входного напряжения как знаковую волну с величиной 10 В и частотой 159 Гц (на частоте среза).

    Анализ переходных процессов с помощью фильтра нижних частот

    Вот и все. На частоте среза амплитуда выходного сигнала в 0,707 раз превышает входную амплитуду. И выходная фаза отстает от входной фазы на 45 градусов. Видите, как прекрасна SPICE?

    Эффект нижних частот на прямоугольной волне

    Ниже приводится симуляция SPICE, показывающая влияние фильтра нижних частот на прямоугольную волну.Нарастающие и спадающие фронты выходного сигнала замедляются и асимптотически приближаются к своим конечным значениям (экспоненциальный спад).

    Влияние фильтра нижних частот на прямоугольную волну

    Пассивный фильтр высоких частот

    Теперь давайте посмотрим на фильтр верхних частот, который пропускает высокочастотные компоненты и задерживает низкочастотные компоненты. Если мы поменяем местами резистор и конденсатор так, чтобы конденсатор теперь был включен последовательно с сигналом, мы преобразуемся в фильтр верхних частот.

    Пассивный фильтр высоких частот

    Передаточная функция:

    На основе передаточной функции это фильтр первого порядка, поскольку нет степени s выше 1.Кроме того, он имеет один ноль при S = ​​0 и один полюс при s = -1 / RC. Частота среза определяется полюсом и поэтому такая же, как для фильтра нижних частот.

    Частота среза =

    Фильтр высоких частот Анализ переменного тока

    Анализ переменного тока с фильтром высоких частот Анализ

    переменного тока с помощью SPICE показывает, что величина передаточной функции мала на низкой частоте и увеличивается до частоты среза 1 / (2 * pi * RC) = 159 Гц, где она выравнивается. Как и ожидалось, фильтр верхних частот ослабляет низкочастотные сигналы и пропускает высокочастотные сигналы.

    Анализ переходных процессов в фильтре высоких частот

    Анализ переходных процессов с помощью фильтра верхних частот

    Анализ переходных процессов входного сигнала, настроенного на синусоидальную волну с частотой, равной частоте среза, дает выходной сигнал, фаза которого предшествует входу на 45 градусов (или запаздывает на 315 градусов), а величина которого составляет 0,707 раза. входная величина.

    Определение, схема, характеристики и применение

    Была эпоха, когда во время телефонных звонков на расстоянии нужно было подносить рот очень близко к передатчику, говорить очень медленно и очень громко, чтобы человек на другом конце провода мог отчетливо слышать сообщение.Сегодня мы даже можем совершать видеозвонки по всему миру с высококачественным разрешением. Секрет такого грандиозного развития технологий заключается в Электрический фильтр теория и Теория линии передачи . Электрические фильтры — это схемы, которые пропускают только выбранную полосу частот, подавляя другие нежелательные частоты. Одним из таких фильтров является фильтр верхних частот .


    Что такое фильтр высоких частот?

    Определение фильтра верхних частот — это фильтр, который пропускает только те сигналы, частоты которых выше, чем частоты среза, тем самым ослабляя сигналы более низких частот.Значение частоты среза зависит от конструкции фильтра.

    Цепь фильтра верхних частот

    Базовый фильтр высоких частот состоит из последовательного соединения конденсатора и резистора . Пока входной сигнал подается на конденсатор, выходной сигнал проходит через резистор.

    Схема

    фильтра верхних частот В этой схеме конденсатор имеет высокое реактивное сопротивление на более низких частотах, поэтому он действует как разомкнутая цепь для низкочастотных входных сигналов, пока не будет достигнута частота среза «fc».Фильтр ослабляет все сигналы ниже уровня частоты среза. На частотах выше уровня частоты среза реактивное сопротивление конденсатора становится низким, и он действует как короткое замыкание на эти частоты, позволяя им проходить непосредственно на выход.

    Пассивный RC-фильтр высоких частот

    Показанный выше фильтр верхних частот также известен как пассивный RC-фильтр верхних частот , поскольку схема построена с использованием только пассивных элементов . Для работы фильтра нет необходимости подключать внешнее питание.Здесь конденсатор является реактивным элементом, а выходной сигнал проходит через резистор.

    Характеристики фильтра верхних частот

    Когда мы говорим о частоте среза , мы имеем в виду точку на частотной характеристике фильтра , где усиление равно 50% от пикового усиления сигнала. 3 дБ от пикового усиления. В фильтре высоких частот усиление увеличивается с увеличением частот.

    Кривая частоты фильтра верхних частот

    Эта частота среза fc зависит от значений R и C схемы.Здесь постоянная времени τ = RC, частота среза обратно пропорциональна постоянной времени.

    Частота среза = 1 / 2πRC

    Коэффициент усиления схемы определяется как AV = Vout / Vin

    . Т.е. AV = (Vout) / (V in) = R / √ (R 2 + Xc 2 ) = R / Z

    При низкой частоте f: Xc → ∞, Vout = 0

    На высокой частоте f: Xc → 0, Vout = Vin

    Частотная характеристика фильтра высоких частот или График Боде фильтра высоких частот

    В фильтре верхних частот все частоты, лежащие ниже частоты среза «fc», ослабляются.В этой точке частоты среза мы получаем усиление -3 дБ, и в этот момент значения реактивного сопротивления конденсатора и резистора будут одинаковыми. R = Xc. Прирост рассчитывается как

    .

    Усиление (дБ) = 20 log (Vout / Vin)

    Наклон кривой фильтра высоких частот составляет +20 дБ / декаду. после прохождения уровня частоты среза выходной отклик схемы увеличивается от 0 до Vin со скоростью +20 дБ на декаду, что составляет увеличение на 6 дБ на октаву.

    Частотная характеристика фильтра верхних частот

    Область от начальной точки до точки частоты среза называется полосой остановки, поскольку пропускать никакие частоты не разрешается.Область сверху от точки частоты среза. то есть точка -3 дБ известна как полоса пропускания . На частоте среза амплитуда точечного выходного напряжения будет составлять 70,7% входного напряжения.

    Здесь ширина полосы фильтра обозначает значение частоты, с которой разрешено проходить сигналам. Например, если полоса пропускания фильтра верхних частот задана равной 50 кГц, это означает, что разрешены только частоты от 50 кГц до бесконечности.

    Фазовый угол выходного сигнала +450 на частоте среза.Формула для расчета фазового сдвига фильтра верхних частот:

    ∅ = arctan ⁡ (1 / 2πfRC)

    Кривая фазового сдвига

    В практическом применении выходной отклик фильтра не простирается до бесконечности. Электрические характеристики фильтрующих элементов накладывают ограничение на отклик фильтра. Правильно подобрав компоненты фильтра, мы можем отрегулировать диапазон частот, которые необходимо ослабить, диапазон, который необходимо пропустить и т. Д.

    Фильтр высоких частот с ОУ

    В этом фильтре верхних частот вместе с пассивными фильтрующими элементами мы добавляем в схему операционный усилитель ОУ .Вместо получения бесконечной выходной характеристики здесь выходная характеристика ограничена характеристиками разомкнутого контура операционного усилителя . Следовательно, этот фильтр действует как полосовой фильтр с частотой среза, которая определяется характеристиками полосы пропускания и усиления операционного усилителя.

    Фильтр высоких частот с использованием операционного усилителя

    Коэффициент усиления операционного усилителя по напряжению без обратной связи действует как ограничение полосы пропускания усилителя. Коэффициент усиления усилителя уменьшается до 0 дБ с увеличением входной частоты.2)

    где Af — усиление полосы пропускания фильтра = 1+ (R2) / R1

    f — частота входного сигнала в Гц

    fc — частота среза

    При использовании резисторов и конденсаторов с низким допуском эти активные фильтры высоких частот обеспечивают хорошую точность и производительность.

    Активный фильтр высоких частот

    Фильтр верхних частот , использующий операционный усилитель , также известен как активный фильтр верхних частот , потому что наряду с пассивными элементами, конденсатором и резистором, в схеме используется операционный усилитель с активным элементом.Используя этот активный элемент, мы можем управлять частотой среза и диапазоном выходного отклика фильтра.

    Фильтр высоких частот второго порядка

    Все схемы фильтров, которые мы видели до сих пор, считаются фильтрами верхних частот первого порядка. В фильтре верхних частот второго порядка дополнительный блок RC-цепи добавляется к фильтру верхних частот первого порядка на входном тракте.

    Фильтр высоких частот второго порядка

    Частотная характеристика фильтра верхних частот второго порядка аналогична фильтру верхних частот первого порядка.Но во втором порядке полоса заграждения фильтра верхних частот будет вдвое больше, чем у фильтра первого порядка при 40 дБ / декада. Фильтры более высокого порядка могут быть сформированы путем каскадирования фильтров первого и второго порядка. Хотя нет ограничений на порядок, размер фильтров увеличивается вместе с их порядком, и точность ухудшается. Если в фильтре более высокого порядка R1 = R2 = R3 и т. Д.… И C1 = C2 = C3 = и т. Д., То частота среза будет одинаковой независимо от порядка фильтра.

    Фильтр верхних частот второго порядка

    Частота среза активного фильтра верхних частот второго порядка может быть задана как

    .

    fc = 1 / (2π√ (R3 R4 C1 C2))

    Передаточная функция фильтра высоких частот

    Поскольку сопротивление конденсатора часто меняется, электронные фильтры имеют частотно-зависимую характеристику.(п-1) + ⋯ + b0)

    Порядок фильтра известен по степени знаменателя. Полюса и нули схемы извлекаются путем решения корней уравнения. Функция может иметь действительные или комплексные корни. То, как эти корни нанесены на плоскость s, где σ обозначается горизонтальной осью, а ω обозначается вертикальной осью, раскрывает много информации о схеме. Для фильтра высоких частот ноль находится в начале координат.

    H (jω) = Vout / Vin = (-Z2 (jω)) / (Z1 (jω))

    = — R2 / (R1 + 1 / jωC)

    = -R2 / R1 (1 / (1+ 1 / (jωR1 C))

    Здесь H (∞) = R2 / R1, усиление при ω → ∞

    τ = R1 C и ωc = 1 / (τ).2) = 0 при ω = 0; H (∞) / √2, когда ω = ω_c;

    и H (∞), когда ω = ∞. Здесь отрицательный знак указывает на фазовый сдвиг.

    Когда R1 = R2, s = jω и H (0) = 1

    Итак, передаточная функция фильтра высоких частот H (jω) = jω / (jω + ω_c)

    Масляный фильтр высоких частот

    Помимо подавления нежелательных частот, идеальный фильтр должен также иметь равномерную чувствительность для требуемых частот. Такой идеальный фильтр непрактичен. Но Стивен Баттер Ворс в своей статье «К теории фильтров-усилителей» показал, что такого типа фильтра можно добиться, увеличив количество фильтрующих элементов правильной величины. n , что при решении равно

    ‘n’ контролирует порядок перехода между полосой пропускания и полосой заграждения.2) для заказа 2

    Следовательно, передаточная функция каскада в фильтре высоких частот равна

    .

    График Боде масляного фильтра верхних частот

    Применение фильтра верхних частот

    Применения фильтра высоких частот в основном включают следующее.

    • Эти фильтры используются в динамиках для усиления.
    • Фильтр высоких частот используется для удаления нежелательных звуков вблизи нижнего края слышимого диапазона.
    • Чтобы предотвратить усиление постоянного тока , которое может повредить усилитель, для связи по переменному току используются фильтры верхних частот.
    • Фильтр высоких частот в обработке изображений : Фильтры высоких частот используются при обработке изображений для повышения резкости деталей. Применяя эти фильтры к изображению, мы можем преувеличить каждую мельчайшую деталь изображения.

    alexxlab

    Добавить комментарий

    Ваш адрес email не будет опубликован. Обязательные поля помечены *